SlideShare ist ein Scribd-Unternehmen logo
1 von 30
Downloaden Sie, um offline zu lesen
สหพั น ธ นิ สิ ต นั ก ศึ ก ษาแพทย แ ห ง ประเทศไทย (สพท.)
        National Council of Thai Medical Students (NCTMS)
         วิทยาลัยแพทยศาสตรกรุงเทพมหานครและวชิรพยาบาล คณะแพทยศาสตร มหาวิทยาลัยขอนแกน               คณะแพทยศาสตร จุฬาลงกรณมหาวิทยาลัย
         คณะแพทยศาสตร มหาวิทยาลัยเชียงใหม             คณะแพทยศาสตร มหาวิทยาลัยธรรมศาสตร          คณะแพทยศาสตร มหาวิทยาลัยนราธิวาสราชนครินทร
         คณะแพทยศาสตร มหาวิทยาลัยนเรศวร                คณะแพทยศาสตร มหาวิทยาลัยบูรพา               คณะแพทยศาสตรพระบรมราชชนก
         วิทยาลัยแพทยศาสตรพระมงกุฎเกลา                คณะแพทยศาสตร มหาวิทยาลัยมหาสารคาม           วิทยาลัยแพทยศาสตร มหาวิทยาลัยรังสิต
         คณะแพทยศาสตรโรงพยาบาลรามาธิบดี                สํานักวิชาแพทยศาสตร มหาวิทยาลัยวลัยลักษณ   คณะแพทยศาสตร มหาวิทยาลัยศรีนครินทรวิโรฒ
         คณะแพทยศาสตรศิรราชพยาบาล
                          ิ                             คณะแพทยศาสตร มหาวิทยาลัยสงขลานครินทร       สํานักวิชาแพทยศาสตร มหาวิทยาลัยเทคโนโลยีสุรนารี
         วิทยาลัยแพทยศาสตรและสาธารณสุข มหาวิทยาลัยอุบลราชธานี
       ๓๑๗/๕ ถนนราชวิถี แขวงทุงพญาไท เขตราชเทวี กรุงเทพมหานคร ๑๐๔๐๐ โทรสาร ๐-๒๓๕๔-๗๗๕๐ ตอ ๑๑๒


                     Exercise for National License Part II
                     CLINICAL SCIENCE : March 2009
                               By NCTMS2009




แนวขอสอบชุดนี้เปนการรวบรวมแนวขอสอบที่สงมาจากโรงเรียนแพทย3แหง ไดแก
(เรียงตามลําดับตัวอักษรภาษาอังกฤษ)
CU คณะแพทยศาสตร จุฬาลงกรณมหาวิทยาลัย
NU คณะแพทยศาสตร มหาวิทยาลัยนเรศวร
PSU คณะแพทยศาสตร มหาวิทยาลัยสงขลานครินทร

รวบรวมและเรียบเรียงโดย ฝายวิชาการ คณะแพทยศาสตร มหาวิทยาลัยนเรศวร
ตรวจทานโดย คณะกรรมการสหพันธนิสิตนักศึกษาแพทยแหงประเทศไทย รุนที่19

หากมีขอผิดพลาดประการใด ขออภัยมาไว ณ ที่นี้
แนะนําหรือติชมไดที่ www.nctms.in.th/webboard
                                                                                                       09/09/09
สหพันธนสิตนักศึกษาแพทยแหงประเทศไทย
                                                                          ิ
Medicine                                             4. ชาย 20ป ไขสูง5วัน ปวดหัว คลื่นไสแตไมอาเจียน
1.ชาย 18y เพื่อนนํามาสง ER เนื่องจากพบวานอน        PE : mild conjunctival injection, no jaundice,
สลบ ปลุกไมตื่น PE : GCS 4, Pupil 1 mm, RR 6         V/S : BT 39.5 C, PR 90, RR 14, BP 120/90
นอกจากชวยหายใจแลวจะรักษาอยางไรตอ                 liver 1 FB RBCM, span 11 cm, petechia on
       A) Naloxone                                   extremities
       B) Atropine                                   CBC : Hct 45% WBC 4,300 N50 L40 M3
       C) Glucose                                            atypL 7 plt 60,000
                                                     จงใหการวินิจฉัย
2. ชาย 20 ป ถูกงูกัด 1 ชม.ที่เทาเดินในสวน ตรวจ             A) leptospirosis
รางกายมี bilateral ptosis , Right foot 2 fnags              B) DHF
mark with swelling ถามวางูอะไรกัด                           C) scrub typhus
       A) งูเหา                                             D) malaria
       B) งูกะปะ                                             E) enteric fever
       C) งูแมวเซา
       D) งูทับสมิงคลา                               5. ชาย 20 ป ไขสูง ปวดหัว
       E) งูเขียวหางไหม                             PE: subconjunctival hemorrhage BT 39 C, stiff
                                                     neck positive, petechia and purpura on both
3. ถาม GCS ชายหลังประสบอุบัติเหตุ ไมลืมตา แต       legs จงบอก organism
เรียกแลวลืมตา บนพึมพํา พูดเปนคําไมรูเรื่อง ยก           A) N. meningitides
มือขึ้นมาปดได ถูกตําแหนงที่ทําใหเจ็บ                     B) K. influenzae
         A) 7                                                C) S. pneumoniae
         B) 8                                                D) S. aureus
         C) 9                                                E) M. pneumoniae
         D) 10
         E) 11                                       6.ชายไข 5 วัน มีคลื่นไส ถายเหลว มีไอเจ็บคอรวม
                                                     ดวย ตรวจรางกายตับโต Lab Pltลดลง Atyp lymph
                                                     เพิ่มขึ้น
                                                     A) DHF




                     Exercise for National License Part II : March 2009 By NCTMS2009
                     หากมีขอเสนอแนะ ติชม เชิญแนะนําไดที่ www.nctms.in.th/webboard                   1
สหพันธนสิตนักศึกษาแพทยแหงประเทศไทย
                                                                            ิ
7.ชาย 30y ไข ป วดศี ร ษะมาก ซึ ม มี purpura,          12.ชายชาวนา บานอยูกาฬสินธุ เปนไข หอบเหนื่อย
petichiae ขึ้นตามตัว stiffness positive เกิดจากเชื้อ   CXR : Patchy infiltration Cr : raising
ใดมากที่สุด                                            U/S : multiple nodular hypoechoic mass
        A) Neisseria meningitides                             A) Melioidosis
        B) Stap aureus
        C) Haemophilus influenzae                      13. หญิง 30 ป มีจุดเลือดออกตามตัวมา 3 สัปดาห
                                                       บางครั้งมีเลือดออกตามไรฟนรวมดวย
8. Blood smear, พบ multiple ring form จะ รักษา         PE: not pale, no jaundice, petechiae purpura at
อยางไร                                                trunk and extremities
        A) cholroquine                                 CBC : Hb 12 Hct 36 MCV 80 WBC 4,600 N65
        B) IV Artesunate+tetracycline                  L30 M5 Plt 36,000
        C) chlroquine+primaquine                       จงหาการรักษา ที่เหมาะสม
        D) artesunate+mefloquine 5 quinine                     A) prednisolone
                                                               B) cyclophosphamide
9.รูปไขพยาธิ Opistorchis viverrini Txยังไง                    C) IVIG
         A) Albendazole                                        D) splenectomy
         B) Mebendazole                                        E) plt transfusion
         C) Praziquantel
                                                       14.ชาย 30y คลําไดLN ที่คอโตทั้งสองขาง มา 3
10.ผูปวย Nephrotic syndrome on prednisolone          เดือน กอนโตขึ้นเรื่อยๆ 1 สัปดาห รูสึกอึดอัดหายใจ
นาน ถามวาเสี่ยงติด Parasite larva อะไร                ลําบาก เสียงดังเวลาหายใจเขา
         A) Strongyloides                              PE : BP 140/90 RR 24 PR 100
                                                       inspiratory stridor, bilateral cervical LN
11.ผูปวยหญิงไปเที่ยวนราธิวาส เปนไขปวดขอ ผล        enlargement 2 cm., lung clear with respiratory
เลือดเปน Lymp เดน, Atyp 15% , Plt ต่ํา               stridor , liver 3 cm. BRCM, spleen 8 cm. BLCM
         A) Lepto                                      จงหาการรักษา เบื้องตน
         B) Scrub                                               A) Beta2 agonist inhale
         C) Chicukunya fever                                    B) Adrenaline SC
                                                                C) Furosemide IV
                                                                D) Dexamethasone IV
                                                                E) ET tube

                      Exercise for National License Part II : March 2009 By NCTMS2009
                      หากมีขอเสนอแนะ ติชม เชิญแนะนําไดที่ www.nctms.in.th/webboard                    2
สหพันธนสิตนักศึกษาแพทยแหงประเทศไทย
                                                                           ิ
15. ชายอายุ 50 ป สูบบุหรี่วันละ 1-2 ซองมา 30 ป       18.หญิง 70ป เปน Alzheimer 4-5ป 1ป กอนไมยอม
สังเกตวามีหนาบวม หายใจแนนติดขัดมา 2 สัปดาห         กินอาหาร ญาติใหอาหารดวยการบีบ syringe เขา
PE : facial swelling, superficial vein dilatation at   ปาก ไมมีอาการสําลัก แตผอมลง
anterior chest wall, Chest –normal, Abd-normal,        PE : generalize wasting, pale, pitting edema
Other-WNL                                              both legs, white nail
จงใหการวินิจฉัย                                       ทานจะเลือกใหอาหารผูปวยอยางไร
        A) Cor pulmonale                                       A) Nasogastric tube
        B) Cardiac tamponade                                   B) Nasoduodenal tube
        C) Nephrotic syndrome                                  C) Nasojejunal tube
        D) Pulmonary TB                                        D) Percutaneous endoscopic
        E) Bronchogenic CA                                        gastrostomy tube
                                                               E) Percutaneous endoscopic
16. ผูปวยชายอายุ 40 ป กินเหลามากเพลีย เปนมา                  jejunostomy tube
2 เดือน CBC : WBC 12,000 มี WBC granulocyte
ทุก series , plt 650,000 splenomegaly จงใหการ         19.ชาย อายุ 40ป สวนสูง 160 cm. นน. 90 kg.
วินิจฉัย                                               FBS 140 mg%---repeat 145 mg% ไมมีอาการของ
         A) acute myelofibrosis                        DM ทานจะทําอยางไร
         B) acute myeloid leukemia                            A) สังเกตอาการ
         C) chronic myeloid leukemia                          B) ควบคุมอาหารและออกกําลังกาย
         D) malignant lymphoma                                C) Insulin injection
                                                              D) กินยากลุม incretin
                                                                           
17.ชาย 35ป นน.137 kg. สูง 170 cm. รอบเอว 137                 E) กินยาเพิ่มการหลัง insulin
                                                                                  ่
cm.เปน HT 2y. BS 120 mg%
สําหรับผูปวยรายนี้จะลดความอวนอยางไรใหไดผล        20. หญิง 48y TB 6/3 SGOT 50 SGPT 50 ALP 500
ระยะยาวที่สุด                                          DB 6/3 U/S hyperecchoic no occupy lesion
        A) Life style modification                     เสี่ยงขาดสารใด
        B) Orlistat                                            A) retinol
        C) Sibutramine                                          B) zinc
        D) Gastric resection                                   C) selenium
        E) GI bypass                                           D) toliem
                                                               E) colbalamine

                      Exercise for National License Part II : March 2009 By NCTMS2009
                      หากมีขอเสนอแนะ ติชม เชิญแนะนําไดที่ www.nctms.in.th/webboard                 3
สหพันธนสิตนักศึกษาแพทยแหงประเทศไทย
                                                                           ิ
21. อาการ Parkinson รักษา                             26. หญิง 40 ป เปนเบาหวาน น้ําตาล 134mg/dL
       A) Levodopa                                    ปวดหูขางขวา บรรยายตรวจรางกายเหมือน Herpes
       B) Benzhexol                                   zoster oticus มีกลามเนื้อใบหนาออนแรงแบบ
                                                      Lower motor neuron จะทําอะไรตอ
22. เปน DM facial palsy ดาน? เหมือน LMN ( คิด               A) CT scan
วาเปน Bell’s palsy ) น้ําตาลไมสูง ถาม Tx                   B) ยากันชัก อะไรซักอยาง คนจดจําไมได
         A) Prednisolone                                      C) Prednisolone
                                                              D) ASA
23. ใหอาการมาประมาณ Myasthenia gravis คือ
Proximal muscle weakness อาการเปนมากชวง             27. หญิง 30 ป มีอาการปวดหัวเปนๆ หาย ๆ 3-4
บายๆ – Proptosis bilateral ถามวาจะสงตรวจอะไร       ครั้งตอเดือน เปนมา 2 ป ตรวจรางกายปกติดี
เพื่อใหได Dx…                                                A) diazepam
          A) Prostigmine test                                  B) amitryptyline
          B) CT brain                                          C) lorazepam
          C) Electrolyte                                       D) CT brain
          D) Muscle biopsy
                                                      28.ชาย 24 ป ชัก 1 hr ชักตั้งแตอายุ 15 ป ชัก 3-4
24. PE พบชาที่นิ้วโปงกลางชี้ , Tinel – negative จะ   ครั้ง/ป ไมไดรักษามา 1 ป ตอนนี้ไมมีอาการ
สงตรวจอะไร                                                     A) diazepam
       A) Cholesterol                                           B) phenyltoin
       B) LDL                                                   C) phenobarb
       C) Cervical traction                                     D) midazolam
       D) Pronator …                                            E) gabapentin

25 คนไข 75 ป ใหอาการมาแบบ Parkinson เลย            29. ชาย 30 ป มาดวยชาตั้งแตหัวไหลไปถึงปลายเทา
PE:Cocwheel rigidity , resting tremor                 ตรวจร า งกายพบ กล า มเนื้ อ กางแขน อ อ นแรง
ใหยา อะไร                                            กลามเนื้อขากระตุกเกร็ง lesion อยูที่ใด
        A) Levodopa                                            A) cerebral cortex
                                                               B) cervical cord
                                                               C) cervical nerve root


                     Exercise for National License Part II : March 2009 By NCTMS2009
                     หากมีขอเสนอแนะ ติชม เชิญแนะนําไดที่ www.nctms.in.th/webboard                   4
สหพันธนสิตนักศึกษาแพทยแหงประเทศไทย
                                                                          ิ
30. ชาย 18 ป MCA สมองบวม ใส ET tube มา 2           34. ชาย 30 ป เหนื่อยมา 2 hr. สูบบุหรี่ครึ่งซอง/วัน
เดือน หลังoff tube มีอาการหายใจขัด มี stridor ทั้ง   มาเป น เวลา15 ป มี อ าการไอ หอบ หอบเวลา
เขาและออก เวลาพูดมีลมออกรั่วออกมา                   กลางคืน+เวลาอากาศเย็น จงใหการวินิจฉัย
จง diagnosis                                                 A) Bronchiectasis
       A) vocal cord granulation                             B) Bronchial asthma
       B) arythnoid dislocation                              C) Chronic bronchitis
                                                             D) Pulmonary TB
31 ชาย 33ป ปวดหัว มาหลายเดือน ใจสั่น เหนื่อย                E) CA bronchus
งาย flushing BP 180/110 PR 110 investigation?
         A) urine VMA                                35. ชายไขต่ําๆ ไอ หอบ มา 2 วัน ฟงปอดมี
         B) CT brain                                 crepitation ดานขวา CXR:RLL infiltration ถามเชื้อ
         C) Thyroid function                                 A) H.influenza
         D) pyelogram                                        B) S.pneumonae
                                                             C) Mycloplasma pnemoniae
32. ผูปวยหญิง อายุ 44 ป สูง 155 cm. BW: 77 kg.            D) Chlamydia
ชอบกินอาหารรสจัด PE: BP สูง PR: 78/min                       E) S.aureus
จงmanagement
          A) ACEI                                    36. หญิง 28 ป ไขต่ําๆ ไอ มีเสมหะ มี bullous
          B) B-blocker                               myringitis คิดถึงเชื้ออะไร
          C) CCB                                             A) S.pneumoniae
          D) diuretic                                        B) H.influenzae
          E) life style modification                         C) RSV
                                                             D) Mycoplasma
33. เหมือนโพย NT 2007 ผูปวยกินเหลา มี HF
จงmanagement
        A) Digoxin
        B) Thiamine




                     Exercise for National License Part II : March 2009 By NCTMS2009
                     หากมีขอเสนอแนะ ติชม เชิญแนะนําไดที่ www.nctms.in.th/webboard                   5
สหพันธนสิตนักศึกษาแพทยแหงประเทศไทย
                                                                            ิ
37. หญิงไมมีอายุ (โจทยไมใหมา) คันที่ขาซายมาก             A) serum Ig A level
PE พบ ill-defined erythematous papulovesicle                  B) serum C 3 complement
confluence to plaque at left leg ใหยาอะไรดี                  C) ANA
       A) antibiotic                                          D) ASO titer
       B) antiviral                                           E) serum cholesterol an albumin
       C) antifungal
       D) antihistamine                                41.หญิง70ป มีไข ปวดเมื่อยตัว1wk กอนหายเอง PE
       E) corticosteroid                               : multiple erythematous subcutaneous nodusum
                                                       ควรสงตรวจอะไร
38. คนไขมีผื่นเปนตุมน้ําตามตัว แขนขา                         A) CXR
PE: discrete vesicle, bullae of trunk, low                      B) UA
extrimities การตรวจใดชวยวินิจฉัย                               C) Plain abdomen
         A) gram stain                                          D) ESR
         B) AFS
         C) Tzack smear                                42.ชายพบผื่นที่ขาหนีบลามชาๆ well define scaly
                                                       annular with arciform erythematous patch สง
39. หญิง 29 ป เปนไข ปวดเมื่อยกลามเนื้อ ไมไดกิน   KOH จะพบอะไร
ยา หายเองได ตอมามี subcutaneous nodule at                   A) Short hyphae with thick oval
leg สงตรวจทางหองปฎิบัติการใด                                B) Pseudohyphae
        A) ESR                                                C) Branching hyphae with spore
        B) x ray                                              D) Septated hyphae with arthrospore
        C) CBC
                                                       43. ผูปวยชายปวดขอเขาขวา มีไข เจาะหนองที่เขา
40. เด็กผูหญิง 15 ป บวมทั้งตัวมา 1 เดือน ปวดขอ      ได 1 ml ยอม gram ได gram+ cocci in cluster
เขา ขอเทา มา มี oral ulcer ที่ upper palate  2      Tx ดวย Cloxacillin ไมดีขึ้น เขาขวาบวมมากขึ้น
สัปดาห ตอมา เหนื่อยมากขึ้น, V/S : BT 37.8°C BP       ควรทําอะไรตอ
140/90 mmHg RR 24 /min, erythematous                            A) ให NSAIDs
maculopapular rash both palm an soles, ESR                      B) ให Ceftriaxone
100 mm/hr, Lab: UA :protein 4+ ,RBC                             C) เปลี่ยนยาเปน Vancomycin
numerous , granular,RBC cast                                    D) ใหยาเดิม แตเจาะเขา (aspirate) หนอง
สงวาสงตรวจอะไรที่ชวยวินจฉัยมากที่สุด
                            ิ                                      ออกใหมากที่สุด

                      Exercise for National License Part II : March 2009 By NCTMS2009
                      หากมีขอเสนอแนะ ติชม เชิญแนะนําไดที่ www.nctms.in.th/webboard                   6
สหพันธนสิตนักศึกษาแพทยแหงประเทศไทย
                                                                                       ิ
44. หญิง 43y มีอาการออนเพลียงาย กลามเนื้อแขน             48. ชายอายุ 45 ป ภูมิลําเนา อ.สังขละบุรี
ขาออนแรง เคยมีอาการบวมที่ขา กินHCTZ 50 mg                  จ.กาญจนบุรี มีอาการเพลีย ออนแรงมา 3-4 เดือน
มาตลอด อาการในผูปวยรายนี้นาจะเกิดจากอะไร                 PE : generalized hyperpigmentation,
       A) hyponatrimia                                      hyperkeratosis at palms and soles,
       B) hypokalemia                                       whiteline at finger nails
       C) hypomagnesia                                      Motor power – proximal III/V, distal IV/V
       D) hypocalcemia                                      Decreased pain and touch sensation
                                                            จะใหการรักษาอยางไร
45.ใหรูปงูม าลัก ษณะเป น สี ดํา สลั บ เหลื อ งเข ม ลาย           A) Dimercaptol
แบบสามเหลี่ ย ม (เป น งู ก ะปะครั บ ) ถามว า จะ                   B) Calcium EDTA
investigate อะไร                                                    C) Penicillamine
         A) VCT                                                     D) Deferoxamine
         B) Pulmonary function test                                 E) Dialysis
         C) Peak flow
                                                            49. ผูปวยหมดสติ พูดออแอ ไมเปนคํา เรียกแลวลืม
46. รูปงูบนใบตอง ( คนไขนํางูมาดวย) สงตรวจอะไร            ตาตื่ น ดี กระตุ น ให เ จ็ บ แล ว ป ด ตํ า แหน ง ที่ เ จ็ บ ได มี
ในผูปวยรายนี้                                             GCS เทาไหร
         A) peak flow                                                 A) 7
         B) VCT                                                       B) 8
         C) BUN/Cr                                                    C) 9
         D) UA                                                        D) 10
                                                                      E) 11
47. ชายอายุ 40 ป ทําอาชีพปลูกผัก ไขมา 5วัน ปวด
ศีรษะมาก มี LN cervical โต 2 ขาง conjunctival              50.นางสาวก.ตองการสมัครเปนพยาบาลจึงมาตรวจ
injection, no jaundice, stiffneck negative CBC :            สุขภาพ ผลปรากฎวา HBsAg +, HBsAb – ผล liver
WBC 10,000 PMN 50% Lymph 50% จงใหการ                       function test ปกติ ควรทําอยางไรมากที่สุด
วินิจฉัย                                                            A) รักษาดวย interferon
         A) melioidosis                                             B) ให HBV vaccine
         B) scrub typhus                                            C) ตรวจ US abdomen
         C) dengue infection                                        D) ตรวจ AFP
         D) viral meningitis                                        E) ตรวจ HBsAg ซ้ําในอีก 6 เดือน

                        Exercise for National License Part II : March 2009 By NCTMS2009
                        หากมีขอเสนอแนะ ติชม เชิญแนะนําไดที่ www.nctms.in.th/webboard                                           7
สหพันธนสิตนักศึกษาแพทยแหงประเทศไทย
                                                                         ิ
51. ชายอายุ 30 ป คลํากอนไดที่คอโตมา 3 เดือน      54. ผูปวยหญิง ซีดมา 2 วัน reticulocyte count 8%
กอนโตขึ้นเรื่อยๆ 1 สัปดาหกอนรูสึกอึดอัด หายใจ   MCV         65      Hct      20% จงใหการวินิจฉัย
ลําบาก มีเสียงดังตอนหายใจเขา PE: BP 140/90                   A) thalassemia disease
PR 100 RR 24, inspiratory stridor, lung                       B) AIHA
clear, matted bilateral cervical neck                         C) G 6 PD deficiency
cnlargement 4-8 cm in diameter จงบอก initial                  D) HS
management
       A) Beta 2 agonist inhaler                    55. ชายอายุ 40 ป บานอยูจังหวัดนราธิวาส มีไขมา
       B) SC adrenaline                             4 วัน ตอมามีผื่นขึ้น 2 วันตอมาเริ่มมีอาการปวดเขา
       C) IV furosemide                             2 ขาง วันนี้มีอาการปวดขอเทาและขอศอกทั้ง 2 ขาง
       D) IV dexamethasone                          ตรวจรางกาย BT 40C BP 100/80
       E) endotracheal intubation                   maculopapular rash at trunk and extremities,
                                                    swelling warm and tenderness with limitation of
52. ชายอายุ 25 ป มาดวยไข ปวดศีรษะ ชักเกร็งที่    movement both knees, ankles and elbows. Hct
หนาและแขนดานซาย LP WBC 12 (L 95%)                40% WBC 3500 (N 45% L50% AL 5%) plt.
protein 65 glucose 45 RBC 10-20 gram stain          45000 โรคใดที่ผูปวยมีโอกาสเปนมากที่สุด
and india ink negative จงใหการรักษา                         A) ricketsial infection
        A) acyclovir                                         B) Chigunkunya infection
        B) antifungal                                        C) acute gouty arthritis
        C) antibiotics                                       D) acute rheumatoid arthritis
        D) antiTB                                            E) disseminated gonococcal arthritis

53. ผูปวย on high dose steroid ไอมีเลือดปน ตรวจ   56. ผูปวยหญิง มี trauma มา แลวมีอาการปวดขอ
sputum เปนดังรูป นาจะเปนตัวอะไร                  เขาได ATB cloxacillin ประมาณวาเปน septic
          A) strongiloid                            knee 2 วันแลวอาการไมดีขี้น จง management
          B) ascaris                                         A) เปลี่ยน ATB
          C) paragonimus                                     B) ATB เดิม แลวทํา drainage ใหมากทีสุด
                                                                                                  ่
          D) enterobius                                      C) เพิ่ม dose ATB
          E) necator



                    Exercise for National License Part II : March 2009 By NCTMS2009
                    หากมีขอเสนอแนะ ติชม เชิญแนะนําไดที่ www.nctms.in.th/webboard                   8
สหพันธนสิตนักศึกษาแพทยแหงประเทศไทย
                                                                              ิ
57. หญิงอายุ 60 ป บานอยูจังหวัดกาฬสินธุ มีไข ไอ   60. ผูปวยชายถูกงูกัดบริเวณขอเทาซาย
เรื้อรังมา 1 เดือน เหนื่อยงายขึ้นมา 2 สัปดาห         PE: ptosis both eyes, swelling at left ankle, fang
PE : BT 39C RR 28 PR 100 BP 140/80                     mark เกิดจากงูชนิดใด
pale conjunctiva, no icteric sclerae, liver just                 A) งูเหา
palpable, positive splenic dullness.                             B) งูเขียวหางไหม
LAB: glucose 260 BUN 50 Cr 4.0                                   C) งูกะปะ
CXR: patchy infiltration both lower lung field.                  D) งูทับสมิงคลา
US abdomen: splenomegaly with multiple                           E) งูทะเล
hypoechoic lesions
ผูปวยนาจะเปนโรคใดมากที่สุด                         61. ผูปวยชายไทยอายุ 55 ป กินเหลามานานและ
          A) Leptospirosis                             ตรวจพบว า เป น ตั บ แข็ ง มา 4 ป มี อ าการปวดแน น
          B) Melioidosis                               ทอ งมา 2 สั ป ดาห ปวดบริ เ วณใต ลิ้ น ป แ ละใต ช าย
          C) Salmonellosis                             โครงขวา ปวดตลอดเวลา ไมสัมพันธกับมื้ออาหาร
          D) disseminated TB                           USG: solid ill-defined mass 7 cm. in diameter at
          E) amoebic liver abscess                     right hepatic lobe การสงตรวจในขอใดชวยในการ
                                                       วินิจฉัยไดดีที่สุด
58. ผูปวยชาย underlying cirrhosis มาดวยอาเจียน               A) CEA
เปนเลือด ตรวจรางกายพบ marked            ascites,              B) CA 19-9
crepitation both lungs                                          C) β-HCG
Hct 35% plt. 60000 APTT,PT prolonged                            D) α-fetoprotein
ควรให blood product ตัวใด                                      E) alkaline phosphatase
          A) PRC
          B) plt. conc.                                62. ผูปวยหญิงอายุ 18ป 3 วันกอนมีผื่นขึ้นที่ขาซาย
          C) FFP                                       เปน ill-defined erythematous papulovesicular
          D) cryoprecipitate                           confined to plaques ควรใช topical agent ตัวใด
          E) cryo-removed plasma                                 A) ATB
                                                                 B) Antifungus
59. หนอไมดอง และเกิดอาการออนแรง เกิดจากสาร                    C) Antivirus
ใด                                                               D) Antiparasite
A) Botulinum toxin                                               E) Steroid


                      Exercise for National License Part II : March 2009 By NCTMS2009
                      หากมีขอเสนอแนะ ติชม เชิญแนะนําไดที่ www.nctms.in.th/webboard                          9
สหพันธนสิตนักศึกษาแพทยแหงประเทศไทย
                                                                         ิ
63. ผูปวยหญิงมาดวยอาการเหนื่อยหอบ                Surgery
CXR : patchy infiltration,                          1.หญิง 30 ป มาดวยกอนที่เตานมขวา PE : cystic
Sputum exam : pleomorphic gram positive             mass 2 cm at right breast, not tender at lump,
cocci. ไดยา Augmentin มา 3 วันอาการไมดีขึ้น       no axillary lymphadenopathy investigation ?
CXRซ้ํา พบpleural fluid                                      A) incision
ผลfluid analysis: wbc 2500 N90% LDH 900                      B) excision
จง ใหการรักษา                                               C) FNA
          A) เพิ่ม aminoglycosides                           D) Reexamination next 6 months
          B) เปลียนเปน carbapenem                           E) Mammogram
          C) ใส ICD และให ATB เดิม
          D) Observe                                2.ชาย 55y ประวัติดื่มสุราเปนประจําและเปนโรคตับ
                                                    แข็งนาน 4y มีอาการแนนทองใตชายโครงขวาและลิน  ้
64. ชายอายุ 20 ป มาดวยปสสาวะสีเขมมา 2 วัน BP    ป นาน 2y ตรวจ U/S Hyperdensity ,ill define
140/100 UA RBC>100/hpf WBC 10/hpf with red          mass,เสนผานศูนยกลาง 7 cm.จะสง lab ใดเพื่อ
cell cast protein 1+ ควรรักษาดวยยาใด               ชวยในการวินิจฉัย.
       A) Norfloxacin                                       A) ALP
       B) Furosemide                                        B) AFP
       C) Prednisolone                                      C) Beta-hCG
       D) Cef-3                                             D) CEA
       E) Sodium nitroprusside                              E) CA19-9

65. หญิงอายุ 70 ป tense bullae with vesicle on     3. หญิง 38 ป มีไข ปวดทอง ตรงสะดือ 4 ชั่วโมง
trunk no mucosal invole ควรตรวจทดสอบดวย            คลื่นไส อาเจียน เบื่ออาหาร ยายไปที่RLQ
อะไร                                                PE: mild tenderness RLQ
       A) Gram stain                                         A) acute appendicitis
       B) KOH                                                B) acute diverticulitis
       C) AFB                                                C) ovarian twist
       D) Tzank smear                                        D) pancreatitis
       E) india ink                                          E) pelvic inflammation



                    Exercise for National License Part II : March 2009 By NCTMS2009
                    หากมีขอเสนอแนะ ติชม เชิญแนะนําไดที่ www.nctms.in.th/webboard                10
สหพันธนสิตนักศึกษาแพทยแหงประเทศไทย
                                                                               ิ
4. ชายอายุ 40 ป มาดวยปวดทองบีบ ๆ มีคลื่นไส ไม         7.ชายอายุ 60ป มีไข ตาเหลือง ตัวเหลือง
มี อ าเจี ย น ยั ง ถ า ยอุ จ จาระได ป กติ เคยผ า ตั ด   PE: liver กดเจ็บโต 3cm. below costal margin
appendectomy เมื่อ 10 ปกอน                               U/S hypoechoic mass 6 cm.at segment 6 with
PE: mild dehydration, moderate abdominal                   intraseptate, multiple gallstone and CBD stone,
distention, no guarding, hyperactive bowel                 mild dilate of intrahepatic duct and common
sound, PR : yellow stool การสงตรวจขอใด                   bile duct
เหมาะสมที่สุด                                                      A) Hepatoma
          A) gastroduodenostomy                                    B) Infect hepatic cyst
          B) ultrasound                                            C) Cholangio CA
          C) RBC scan                                              D) Amebic liver abscess
          D) plain film abd. supine and upright                    E) Pyogenic liver abscess
          E) barium enema
note: ควรอานเรื่องแนวทางการ management gut                8. ชายโดนแทงหนาอกซายใตราวนม RR 24 PR 98
obstruction                                                BP 112/60, decreased breath sound Lt. lung,
                                                           trachea in midline ใส ICD แลวมีลมออกมา หลัง
5. ชาย 30 ป ผาตัด appendectomy 3 วันตอมา มี             จากทํา CXR พบวามี pneumothorax 30% จะทํา
ไข ปวดทอง                                                อยางไรตอไป
PE mild tenderness, แผลแดง มี fluctuation                          A) observe อยางใกลชิด
จะทําอยางไรตอไป                                                  B) revise ICD
        A) ตัดไหม                                                  C) set OR emergency for thoracotomy
        B) ตัดไหม กิน ATB                                          D) ใส ICD เพิมอีก 1 เสน
                                                                                 ่
        C) ตัดไหม แหวกแผล                                          E) เช็คระบบ ICD
        D) กิน ATB
        E) ทา ATB                                          9. ผูปวยใสทอชวยหายใจ ผาตัด thyroid postop
                                                           แพทยไปเยี่ยมผูปวยพบหายใจดี แตเมื่อพูดมีเสียง
6. แมบาน รูสึกชานิ้วโปง ชี้ กลาง บางครั้งเจ็บจนตื่น    ออกมาดวย จงใหการวินิจฉัย
กลางดึก ควรรักษาอยางไร                                             A) Vocal cord paralysis
        A) surgery                                                  B) Vocal cord edema
        B) steroid injection                                        C) Arrythenoid dislocation
        C) cervical traction                                        D) Vocal cord granuloma


                       Exercise for National License Part II : March 2009 By NCTMS2009
                       หากมีขอเสนอแนะ ติชม เชิญแนะนําไดที่ www.nctms.in.th/webboard                    11
สหพันธนสิตนักศึกษาแพทยแหงประเทศไทย
                                                                           ิ
10. ผูปวยหญิงกินยาลางหองน้ํา ถาม complication             A) ไมตองทําอะไร
ระยะยาวคืออะไร                                                B) ให rabies vaccine ที่ day 0
          A) Esophageal stricture                             C) ให rabies vaccine ที่ day 0,3
          B) Mucolitis(มั้ง)                                  D) ให rabies vaccine ที่ day 0,3,7
          C) Esophagitis                                      E) ให rabies vaccine ที่ day 0,3,7,14,28
          D) อรอยติดใจ...กินเอง
                                                      Ortho
11. ผูปวย trauma PE: BP drop , neck vein            1. ผูปวยชายอายุ 25 ป ขับรถจักรยานยนตมา 1
engorge, trachea shift to the left , decrease         ชั่วโมง ตรวจรางกายพบ 10 cm. laceration wound
breath sound ขอใดที่จําเปนตองรีบใส ICD มาก        at right leg with tibial fracment exposure ทําการ
ที่สุด                                                ลางแผลและ debridement หลังจากนั้นจะทําการ
         A) neck vein engorge                         รักษาอยางไร
         B) trachea shift, decrease breath sound               A) casting
         C) BP drop                                            B) external fixation
                                                               C) internal fixation
12. ผูชายอายุ 40 ป มีหนองไหลที่ขางรูกนมา 3                 D) suture wound
เดือน ไมไดรักษา ตอมาหนองแตกเอง กินยา ATB                    E) skeletal traction
เอง จากนั้นอีก 3 เดือน บริเวณรอบกนมีตุมขนาด 0.5
cm ที่บริเวณเดิม มีรูเปด มีหนองและเลือดไหล           2. ผูชายอาชีพชางไม complain เรื่องปวดศอกขวา
ออกเปนครั้งคราว กินยาแลวไมดีขึ้น จงวินิจฉัย        ตรวจรางกาย tender at Rt. lateral epicondyle of
       A) hydradenitis suppurativa                    humerus ควรทํา test ใดเพื่อการวินิจฉัย
       B) anal fissure                                        A) Tinel test at forarm
       C) anorectal abscess                                   B) Phalan test
       D) herpes simplex                                      C) resist wrist extensor test
       E) fistula in ano                                      D) resist pronation

13. เด็กหญิงอายุ 6 ปถูกสุนัขกัดที่เทาเปนแผลยาว 2
ซม.เคยไดรับวัคซีนพิษสุนัขบาครบเมื่อ 1 ปกอน ตอง
ใหการดูแลรักษาอยางไร




                     Exercise for National License Part II : March 2009 By NCTMS2009
                     หากมีขอเสนอแนะ ติชม เชิญแนะนําไดที่ www.nctms.in.th/webboard                   12
สหพันธนสิตนักศึกษาแพทยแหงประเทศไทย
                                                                           ิ
3. ผูปวยหญิงอายุ 50 ป ตกจากที่สูง มีแขนผิดรูป มี   5. ผูชาย เลนฟุตบอล ลม เจ็บหัวไหล พบ neck of
อาการปวดแขน บวมกระดกขอมือไมได                      humerus fx กลามเนื้อใดจะเสีย
PE: deformity of arms with crepitus ,                         A) Triceps
dorsiflex grade: 0 ,                                          B) Biceps
finger extension grade : 0 ,                                  C) Deltoid
decrease sensation at dorsum at 1st web                       D) Coracobrachialis
space of hand ,                                               E) Brachilis
Film : oblique middle 1/3 fracture of right
Humerus                                               6. ชาย อายุ 63ป มีอาการปวดสะโพกราวลงขา ปวด
จง management                                         มากขึ้นเมื่อเดินไดระยะ 300 m. ตองพักประมาณ 5
         A) ใสเฝอกแขนทอนบนและทอนลาง              นาที อาการดีขึ้นจึงเดินตอไดมา 6 เดือน 1 เดือนตอ
         B) ทํา EMG และใสเฝอกแขนทอนบนและ           อาการเปนมากขึ้น เดินได 100 m. PE : Loss lumbar
            ทอนลาง                                  lordotic มี tender at lumbar จากการ Dx คิดถึงโรค
         C) ผาตัดดามกระดูกและตอเสนประสาท           ใดมากที่สุด
         D) ผาตัดดามกระดูกและใสเฝอกทอนบน                  A) Spinal instability
            และทอนลาง                                       B) Spinal stenosis
         E) ผาตัดตอเสนประสาทและใสเฝอกแขน                 C) Spondylosis
           ทอนบน                                             D) Chronic arterial occlusion
                                                              E) Protuse nucleus proposus
4. ชายปวดขอศอกขวาดาน lateral epicondyle of
humerus จะตรวจรางกายอะไรเพื่อยืนยันการDx             7. ชายอายุ 17 ป ใหประวัติวา 1 สัปดาหไดรับ
      A) Yegason’s test                               อุบัติเหตุที่เขาขวาจากการเลนฟุตบอล 4 วันกอนมา
      B) Valgus testของขอศอกขวา                      รพ. มีไขและเขาบวม เมื่อวานปวดเขามากจนเดินกระ
      C) Tinel testของทั้งแขนขวา                      เผลก จะสง investigation อะไรที่เหมาะสมที่สุด
      D) ทํา hyperextension of Rt.wrist                        A) ASO titer
      E) ตรวจ sensory ที่ first web space                      B) hemoculture
                                                               C) X-rays
                                                               D) arthrocentesis
                                                               E) uric acid analysis



                     Exercise for National License Part II : March 2009 By NCTMS2009
                     หากมีขอเสนอแนะ ติชม เชิญแนะนําไดที่ www.nctms.in.th/webboard                  13
สหพันธนสิตนักศึกษาแพทยแหงประเทศไทย
                                                                              ิ
8. หญิงอายุ 80 ป หกลมในหองน้ํา film พบ                12. ผูหญิง 63 ป ปวดหลังราวลงขา 2 ขางมา 6
intertrochanteric       fracture   จะทําอย า งไร        เดือน เดินไดประมาณ 300 เมตรจะเริ่มมีอาการ ตอง
        A) surgery                                       นั่ง ยองๆพัก อาการจะดีขึ้น อาการแยลงเรื่อยๆ เดิน
         B) long leg cast                                ไดเหลือ 100 เมตรก็ตองหยุดพัก ตรวจรางกาย
         C) ใหนงรถเข็น
                ั่                                       loss of lumbar lordosis, otherwise normal
         D) traction                                     จง ใหการวินิจฉัย
         E) ประเมินสภาพผูปวยกอนเลือกวิธีการ
                                                                 A) lumbar spine instability
            รักษา                                                 B) lumbar stenosis
                                                                  C) spondylolisthesis
9. ผูปวยชาย ปวดแขนและขอมือขวา ตรวจรางกาย                      D) arterial insufficiency
มี weak abductor pollics brevis มีอาการชาที่ 3
นิ้วครึ่งดาน radial side ทํา Tinel test negative at     13. ผูปวยชาย ถูกรถชนที่หนาแขง ตรวจรางกายมี
wrist ควรทํา test อะไรตอ                                แผลขนาด 1cm คลํา dorsalis pedis และposterior
           A) resist wrist extension test                tibial pulse ไมได Doppler study ไม พบ flow
           B) resist wrist flexion test                  X-ray พบ comminuted fracture and posterior
           C) resist pronation test                      displacement of tibia ถามวาเปน open fracture
           D) resist supination test                     class ใด
                                                                  A) class I
10. What is the most common of low back pain?                     B) class II
       A) spondylosis                                             C) class IIIa
       B) spondylolysis                                           D) class IIIb
       C) loss of lumbar lordosis                                 E) class IIIc
       D) spinal instability
       E) weak back muscle                               Rehab
                                                         1. หญิงปวดหลัง 5 เดือน ราวไปขา SLRT negative,
11. แมบานมีอาการปวดแขนขวา ขอมือขวา อาการ              lost sense S1 นอกนั้นปกติ ทําอะไร
เป น มากขึ้ น ตอนกลางคื น จนสะดุ ง ตื่ น มี อ าการชา           A) ใสรองเทาเสริมสน
ที่นิ้วโปง นิ้วชี้ นิ้วกลาง การรักษาที่เหมาะสมคืออะไร           B) abdominal strengthening exercise
           A) cervical traction                                  C) back strengthening exercise
           B) steroid injection
           C) warm pack

                      Exercise for National License Part II : March 2009 By NCTMS2009
                      หากมีขอเสนอแนะ ติชม เชิญแนะนําไดที่ www.nctms.in.th/webboard                    14
สหพันธนสิตนักศึกษาแพทยแหงประเทศไทย
                                                                            ิ
Eye                                                    ENT
1. มีอาการปวดตาทันที ประมาณวา AACG มี                 1. ผูปวยบานอยูใกลสนามบิน มาดวยหูอื้อ ตรวจ
ประวัติโรค asthma แพ sulfa management อยางไร         Weber test : no lateralized , Renne test AC>BC
        A) atropine                                    , Audio : เสียที่ 4000Hz จงใหการวินิจฉัย
        B) b-block                                             A) Acoustic trauma
        C) alpha 2 agonist                                     B) Meneire
        D) acetazolamide                                       C) Atosclerosis

2. ผูหญิง 58 y เปน DM ตาขวามัวลง เห็นเหมือน          2. ชายไทย ขึ้นลง นั่งเครื่องบิน มีอาการปวดหู
อะไรลอยไปลอยมา PE : Rt retinal hemorrhage ,            PE : hemotympanum จะรักษาอยางไร
exuate , cotton wool spot, neovascularization at              A) Decongestant
disc จะmanage ยังไง                                           B) Tympanectomy
         A) laser
                                                       3. คนงานโรงงานเหล็กเสน มีอาการหูเสื่อมจากเสียง
3. complication ของ traumatic hyphema คืออะไร          ดัง การรักษาที่ตองทําทุกราย คือ
       A) strabismus                                           A) ใหคนหูเสื่อมออกจากงาน
       B) glaucoma                                             B) ใสเครื่องปองกันเสียงขณะทํางาน
       C) ambyopia                                             C) ยายไปแผนกที่ไมมีเสียงดัง
       D) conjunctivitid
                                                       4. ผูปวยมาดวยอาการหูอื้อขางขวา
4. ผูปวยชายไมมีโรคประจําตัว มาดวยอาการตาแดง        Weber: lateralization to the left
ไดประวัตวาไปวายน้ําเมื่อวันกอน เชื้อใดนาจะ เปน   Rinne: AC>BC both ears
สาเหตุมากที่สุด                                        จงใหการวินิจฉัย
          A) Naegeria foweri                                     A) Otoscleriosis
          B) Pseudomonas                                         B) Tympanoscleriosis
          C) H. influenza                                        C) Otitis media wit effusion
          D) S. aureus                                           D) Osiicular chain disruption
                                                                 E) SNHL




                      Exercise for National License Part II : March 2009 By NCTMS2009
                      หากมีขอเสนอแนะ ติชม เชิญแนะนําไดที่ www.nctms.in.th/webboard                  15
สหพันธนสิตนักศึกษาแพทยแหงประเทศไทย
                                                                              ิ
5.ผูปวยเด็กมาดวยอาการปวดหู หลังเปนหวัด รักษา         Radio
ac otitis media ดวย Amoxicillin แลวไม หายใน 2         1. Sphenoid sinus เห็นไดชัดในภาพถายทาใด
วัน ควรรักษาอยางไร                                             A) lateral view
         A) Switch to Augmentin                                 B) Towne view
         B) Switch to Caebapenenm                               C) straight PA view
         C) Add Aminoglycosides                                 D) Inclined PA view
         D) Myringotomy                                         E) Water view

6. ผูปวยมาดวยปวดหู ขยับหูไดลดลง มีอาการหนา          OB & Gyn
เบี้ยวขางเดียวกัน ตรวจพบ external auditory canal        1.หญิง25y GA 39wk เจ็บครรภคลอด
บวม มี non mucous plug with granulation tissue           PE : FH ¾ เหนือ umbilicus
จงใหการวินิจฉัย                                         PV : Cx 3cm. eff 50% ตรวจคลําพบคางเด็กอยู
         A) Cholesteatoma                                ดานหลังซายของมารดา ถาม fetal position
         B) Malignant otitis externa                        A) right mento posterior
                                                            B) left mento posterior
7. ผูปวยชายอายุ 18 ป เกิดอุบัติเหตุจําเปนตองใส        C) right occiput posterior
ET tube นาน 10 วัน จึง off ETT 2 weeks ตอมา                D) left occiput posterior
ผู ป ว ยมาด ว ยอาการเหนื่ อ ยง า ย ตรวจร า งกายพบ      E) mento posterior
bilateral stridor จงใหการวินิจฉัย
           A) Subglottic stenosis                        2. หญิง GA (term แลว ) เจ็บครรภคลอด ไดรับการ
           B) Unilateral vocal cord paralysis            drip oxytocin ตอมาปวดทอง พบรอยคอด 2/3 เหนือ
           C) Intubation granuloma                       หัวเหนา management?
                                                             A) emergency c/s
8.หลังผา thyroid ผูปวยเกิดอาการเสียงแหบ พูด               B) oxytocin dosage reduction
แลวมีเสียงลมออกมา จงใหการวินิจฉัย
         A) Subglottic stenosis                          3. ถาม dose postcoital pill หลังมี SI 2 hr
         B) Unilateral vocal cord paralysis                A) estradiol 60mg, next 60mg in the next 12 hr
         C) Intubation granuloma                           B) estradiol 100mg,next 100mg in the next 12hr
                                                           C) estradiol 100mg, next 60mg in the next 12hr




                      Exercise for National License Part II : March 2009 By NCTMS2009
                      หากมีขอเสนอแนะ ติชม เชิญแนะนําไดที่ www.nctms.in.th/webboard                    16
สหพันธนสิตนักศึกษาแพทยแหงประเทศไทย
                                                                        ิ
4.ผูปวยหลังคลอด มี Bleed 1500 cc.                 8.หญิงทองแรก GA 40 week cervix เปด .. cm
PE : Ut. Floppy , มีเลือดบริเวณ Vg                  effacement 50 % station -1 ischial spine หางกัน
จง Dx                                               11 cm , intertuberous difference 8 cm ตอมาอีก
         A) Ut atony                                2 hr ตรวจภายในยังเหมือนเดิม สาเหตุของการติดขัด
         B) Coagulopathy                            ของการคลอด คือ
         C) Bleed from varicose vein at Vg                  A) pelvic inlet
         D) Vg hematoma                                     B) midplane
         E) Retain placenta                                 C) pelvic outlet
                                                            D) inlet+mid
5. หญิง G1P0 GA 8 wkมีเลือดออทาง Vg 2 วัน มีใจ              E) mid+outlet
สั่น อาเจียนมาก คลื่นไส BP 140/90 PR 110 RR
16 คลําไดยอด Ut สูงกวา suprapubic                 9. หญิง 30 ป GA 34 week มีอาการปสสาวะลําบาก
         A) Thyrotoxicosis                          และแสบรอนบริเวณชองคลอด
         B) Throphoblastic                          PE : vesicle& shallow ulcer at valva
         C) Twin                                    จงใหการวินิจฉัย
         D) Preeclampsia                                    A) syphilis
         E) Preg with myomauterie                           B) herpetic vulvitis
                                                            C) Lipsmutz
6. หญิง Preg G2 มี second half bleeding มี                  D) molluscum contagiosum
เลือดออกทาง vagina เปน painless bleeding จะ
ทําอะไรตอ                                          10.ชาย 40ป หญิง 38ป มาปรึกษาแพทยดวยเรื่องลูก
       A) ultrasound                                คนกอนมีพัฒนาการชา ควรทําอยางไร
                                                           A) ตรวจ chromosome
7. หญิง Preg มาฝากครรภ ตรวจ Hb เปน AE สามี
เปน AA2 (A2:3%, F: 1%) จะmanagementอยางไร         11. หญิง 20 ป ปวดทองนอย มีเพศสัมพันธครั้งแรก
       A) U/S                                       เมื่อ 1 วันกอน UA:WBC 25cell , RBC 3-5cell
       B) ทํา amniocentesis                         กลไกการเกิดโรค
       C) ทํา cudocentesis                                  A) Ascending urethral transmission
       D) ใหเหล็กกินวันละ 3 เวลา                           B) Descending urethral transmission
       E) ฝากครรภตามปกติ                                   C) homogenous spreading


                    Exercise for National License Part II : March 2009 By NCTMS2009
                    หากมีขอเสนอแนะ ติชม เชิญแนะนําไดที่ www.nctms.in.th/webboard                17
สหพันธนสิตนักศึกษาแพทยแหงประเทศไทย
                                                                          ิ
12. แมอายุ 48 ป พออายุ 50 ป มีลูกผิดปกติทาง      16. หญิงตั้งครรภมารพ.ดวยอาการปวดกลามเนื้อ มี
โครโมโซม chromosome 44+XX, 21 trisomy พอ            รอยฟกช้ําและจ้ําเขียวที่เบาตา หนาอก แขน ทอง
แมอยากมีลูกคนที่ 2 ทองตอไปแนะนําอยางไร           และตนขา ทั้งรอยเกาและใหม คิดวาเกิดจาก สาเหตุ
       A) ฝากครรภตามปกติ                            ใดมากที่สุด
       B) ตรวจโครโมโซมเด็กทารก                              A) ITP
       C) แนะนําผสมเทียมโดยใชอสุจิผูอื่น                  B) coagulopathy
       D) แนะนําทํา vitro fertilization โดยใชไข           C) liver disease
          ของผูบริจาค                                      D) domestic violence
       E) ไมตองตั้งครรภ
                                                     17. แมมีหมูเลือด Rh negative พอเปน Rh positive
13. 47,XX+21จะใหคําแนะนําอยางไรกับผูปกครอง        หากตั้งครรภควรสงตรวจอะไรเพื่อเปนการลด โอกาส
        A) หามตั้งครรภ                             ในการเกิด hydrops fetalis
        B) ตั้งครรภืไดแตตองทํา amniotomy                  A) indirect Coombs’ test
                                                             B) direct Coombs’ test
14. หญิง 50 ป postmenopausal bleeding,                      C) anti-D titer
TVS: endometrial thickness 2 mm จะทําอะไรตอ
      A) endometrial biopsy                          18. หญิง 27 ปตั้งครรภครบกําหนด EFW 3200 g
      B) F/C                                         เบงคลอดมา 1 ชั่วโมง PV พบวา fully dilated,
                                                     station+1, mento-posterior position ตองทํา
15. หญิงตั้งครรภ 20 สัปดาห มี greenish yellowish   อยางไรตอ
vaginal discharge, vaginal แดง cervix นุมสีมวง             A) manual rotation
จงใหการรักษา                                                B) F/E
       A) oral doxycycline                                   C) V/E
       B) oral tinidazole                                    D) normal Vg delivery
       C) oral clotrimazole
       D) oral fluconazole                           19. ผูปวยหญิง S/P C/S post op day 4 มีไข ตรวจ
       E) vaginal clotrimazole                       รางกายพบรอยแตกที่รูไหม มี fluctuation จะทําอะไร
                                                               A) ตัดไหมออก
                                                               B) ตัดไหมออกและให ATB
                                                               C) ตัดไหมออกและแหวกแผลลึก
                                                               D) ATB alone

                     Exercise for National License Part II : March 2009 By NCTMS2009
                     หากมีขอเสนอแนะ ติชม เชิญแนะนําไดที่ www.nctms.in.th/webboard                 18
สหพันธนสิตนักศึกษาแพทยแหงประเทศไทย
                                                                               ิ
20. หญิงวัยรุนมาตรวจเรื่องโดนขมขืน ตรวจรางกาย        23. คนทอง term ถูก augmentation ดวย
พบ old tear ที่ 6 นาฬิกา มี sperm ไมมีหาง ปริมาณ       syntocinon 20d/min ตอมาพบวามีอาการเจ็บครรภ
เล็กนอยที่ posterior fornix มีรอย kiss bite ที่ลําคอ   มาก ตรวจร า งกายพบรอยคอดบริ เ วณ มดลู ก
จงใหความเห็น                                           เ ห ตุ ก า ร ณ ใ ด จ ะ เ กิ ด เ ป น ลํ า ดั บ ต อ ไ ป
        A) ตรวจพบหลักฐานวาโดนขมขืน                             A) uterine rupture
        B) ตรวจพบหลักฐานวาผานการมีเพศ                          B) abruptio placenta
            สัมพันธ                                             C) retained placenta
        C) ตรวจพบหลักฐานวาผูปวยถูกกระทํา
                                                                D) uterine inertia
             ชําเรา                                              E) uterine eversion
21. หญิงวัยรุนทองแลวมีเลือดออก มาพบแพทย
ตองการใหทําแทง เนื่องจากเคยตรวจกับแพทยทา           24. จากขอที่แลว จงใหการรักษาเบื้องตน
นอนแลวแพทยบอกวาทารกสุขภาพไมแข็งแรง ทาน                    A) off syntocinon
จะทําอยางไร                                                   B) set C/S emergency
        A) ทําแทงให เนื่องจากทารกในครรภไม                  C) manual removal of the placenta
             แข็งแรง                                           D) increase syncinon level
        B) ทําแทงให เนื่องจากถาไมทําก็คงมีคน                E) ARM with C/S emergency
             ทําใหอยูดี
        C) ไมทําให เนื่องจากไมมีขอบงชี้ในการยุติ
                                      
             การตั้งครรภทางการแพทย

22. จากการตรวจภายในเพื่อประเมินชองทางการ
คลอด ได diagonal conjugate 11 cm,
Interspinous 9cm, intertuberous 8cm, subpubic
angle 105 มีความผิดปกติที่ใด
        A) pelvic inlet
        B) midplane
        C) pelvic outlet
        D) pelvic inlet + midplane
        E) midplane + pelvic outlet



                      Exercise for National License Part II : March 2009 By NCTMS2009
                      หากมีขอเสนอแนะ ติชม เชิญแนะนําไดที่ www.nctms.in.th/webboard                         19
สหพันธนสิตนักศึกษาแพทยแหงประเทศไทย
                                                                         ิ
Pediatric                                           4. เด็กหอบเหนื่อยตรวจรางกายพบ neck vein
1. เด็ก 10 ป เปนโรคหืด เปนๆ หายๆ รักษาไม        engorge, lung = fine crepitation at base both
สม่ําเสมอ วันนี้หอบเหนื่อยมากขึ้น PE : good         lung มีขาบวม จําไมไดแลว
consciousness,no cyanosis, prolong expiration               A)heart failure
time with wheezing both lungs ไดรับการรักษา
เปน nebulized salbutamol ไป 2 dose แลวอาการ       5. เด็ก 3 ป ปวดทอง PE:Confused , diaphoretic
ยังไมดีขึ้น ควรทําอะไรตอ                          profound sweating , drooling , meiosis ถามวาให
           A) nebulized adrenaline                  การรักษาอยางไร
           B) aminophylline infusion                         A) Naloxone
           C) IV corticosteroid                              B) Atropine
           D) nebulized corticosteroid                       C) Ethanol
           E) nebulized ipratropium bromide                  D) Flumazil
                                                             E) Diphenyramide
2. เด็กทารก อายุ 2 วัน คลอดที่บาน มารพ.ดวย
อาการตัวออน ไมรอง
                                                   6. เด็ก 8 ป มีไข คลื่นไส อาเจียน ปวดทองมา 3วัน
PE : flaccid, flobby, RR 10/min, HR 50/min การ      PE: epigastrium pain
รักษาใดควรทําเปนอันดับแรก                          Lab: serum : Na 145 K 3.0...
        A) Glucose                                  UA: ketone 4+
        B) Arenaline                                ถามวาควรทําอยางไรในชั่วโมงแรก
        C) NSS                                              A) 0.45 % NaCl 20 ml/kg
        D) Chest compression                                B) 0.9% NaCl 20 ml/kg
        E) PPV                                              C) 0.45% NaCl 10 ml/kg + KCl 20 mEq
                                                            D) 0.9% NaCl 10 ml/kg + KCl 20 mEq
3.เด็กอายุ 6y โดนสุนัขไมมีเจาของกัด ปกติไดรับ            E) RLS
vaccine ครบมาตลอดครั้งสุดทายตอน 5y จะให
vaccine อะไร                                        7. เด็กชายอายุ 5 ป มาดวยกลามเนื้อออนแรง
        A) ไมให                                   เปนมากบริเวณ deltoid and hip flexor เกิดจาก
        B) rabies day 0,3                           สาเหตุใด
        C) rabies day 0,3,7                                A) beri beri
        D) rabies day 0,3,7,30                             B) muscular dystrophy
        E) rabies vaccine + TT

                    Exercise for National License Part II : March 2009 By NCTMS2009
                    หากมีขอเสนอแนะ ติชม เชิญแนะนําไดที่ www.nctms.in.th/webboard                 20
สหพันธนสิตนักศึกษาแพทยแหงประเทศไทย
                                                                                ิ
8. เด็ก 2 ป ซีด มีจุดแดงที่ขา มา 2 wks ตรวจ                      A) สงตรวจการไดยิน
รางกายพบ moderate pale , petichiae at leg and                    B) สง Consult หมอตา
RUQ mass about 3x5 cm. firm, hard                                 C) สงแพทยเฉพาะทางทันที
consistency CBC: Hct 21%, plt 15,000, WBC                         D) แกไขภาวะโภชนาการกอนแลวจึงRefer
3,600 มี N 30%, L 50% ,M 6% จง diagnosis                          E) คอยมาผาตัดตอนอายุครบ 1 ป
         A) wilm tumor
         B) neuroblastoma                                 11. เด็ก 5 ป มีอาการเหมอลอย เปนพักๆ ตรวจพบ
         C) hepatoblastoma                                3 Hz. spike wave เมื่อทํา hyperventilation จะให
         D) non-Hodkin lymphoma                           ยาใด
         E) acute lymphoblastic leukemia
                                                          12. เด็กหายใจเสียงดัง ดีขึ้นเวลานอนคว่ํา
9.เด็กหญิง 2ขวบ Wt แรกคลอด 2800g. long                    จง diagnosis
47cm. มาที่ Well baby clinic ผูปวยแข็งแรงดี ตรวจ                A) larlyngomalacia
การเจริญเติบโตตาม Growth curve อยูที่ P3 ทั้ง                    B) epiglottic cyst
สว นสู ง และน้ํ า หนั ก พ อสู ง 156cm. แม สูง 149cm.
แมมีประจําเดือนครั้งแรกตอน 13y ทานจะทําอยางไร          13. เด็กชายไทยอายุ 7 ป มีอาการไข น้ํามูกสีเขียวขน
          A) เปนภาวะปกติเพราะเปนผลจาก                   ไหล มีอาการเสมหะเขียวเปนๆหายๆ
          พันธุกรรม                                       PE : erythematous swelling turbinate,
          B) ใหกินอาหารที่มีโปรตีนและพลังงานสูง          postnasal drip, mucopurulent discharge
          C) สงปรึกษากุมารแพทย                          ผูปวยรายนี้นาจะเปนอะไรมากที่สุด
          D) ตรวจระดับ hormone                                     A) Sinusitis
          E) ตรวจ bone mass                                        B) Bronchiolitis
                                                                   C) Allergic rhinitis
10. ผูปวยเด็ก 5 เดือน มาดวยมีกอนที่คิ้วขวา ใหญ
มาบดบังตาขวา เด็กยังชันคอไมได เรียกแลวไมหัน           14. เด็กแรกเกิดดูดนมแลวเขียว หายเขียวเวลารอง
X-ray พบวาไมมี Frontal sinus และกระดูก Frontal          จงใหการวินิจฉัย
เชื่อมกับ Ethmoid , Body weight – P25 ควรทํา                     A) Coanal atresia
อยางไรตอ                                                       B) TE fistular
                                                                 C) PDA



                       Exercise for National License Part II : March 2009 By NCTMS2009
                       หากมีขอเสนอแนะ ติชม เชิญแนะนําไดที่ www.nctms.in.th/webboard                      21
สหพันธนสิตนักศึกษาแพทยแหงประเทศไทย
                                                                               ิ
15. เด็ก 4ป ไข ไอ หอบ กอนหนานี้มีน้ํามูกใสๆไหล               A)   Arthrocentesis
PE:Trachea shift to Left , decrease Rt.breath                    B)   Rt. Knee x-ray
sound CXR:hazzeness Rt.lung                                      C)   Bone aspiration
ควรสงตรวจอะไรเพื่อชวย Dx.                                      D)   ESR
       A) Tuberculin test                                        E)   Bone scan
       B) …
       C) Thoracocentesis                                20. เด็ก 17 ป ประสบอุบัติเหตุกระแทกขณะเลน
       D) …                                              ฟุตบอล 1 สัปดาหกอนมารพ.
       E) Nasopharyngeal culture                         4 วันกอนมารพ. ปวดเขา มีไข
                                                         1 วันกอนมารพ. ปวดจนเดินลงน้ําหนักไมได
16. เด็กแรกคลอดอาเจียนเปนเลือด หลังคลอด 6 hr            จะสงตรวจอะไรเพิ่มเติม
        A) Apt’s test                                            A) Film knee
        B) striae tear                                           B) Hemoculture
        C) plt                                                   C) Arthrocentesis
                                                                 D) Serum uric acid
17.เด็กชายมาดวย multiple purpura ที่ลําตัว แขน
ขา และรอยเตารีด พอบอกวาลมลงโดนเตารีด ควร              21. เด็ก 12 ป นน.ขึ้นผิดปกติตั้งแตอายุ 2 ป
ทําอยางไร                                               PE : obesity, acanthosis nigricans
        A) ซักจิตเวชในครอบครัว                           นาจะตรวจพบสิ่งใด
        B) สง coagulogram                                      A) Cortisol สูง
        C) แยกจากพอแม แลวคอยซักประวัติ                      B) Hyperinsulinemia
                                                                C) Growth hormone สูง
18. ผูปวยเด็กมี acanthrosis nigrican ขอใดถูก                 D) Hypothyroidism
          A) insulin เพิม
                        ่                                       E) Hyperuricemia
          B) cortisol เพิม่
          C) thyroid เพิ่ม                               22. เด็กชายมาดวยอาการเจ็บหู ไดรับการวินิจฉัย
                                                         เปน Otitis media ไดยา Amoxycillin มาทาน 2 วัน
19. เด็กชาย 10 ป ปวดเขาขวา มีไข มา…วัน                อาการไมดีขึ้น ยังเจ็บหู จะ management อยางไร
PE : BT 39 c, swelling at Rt. Knee and proximal                  A) oral amoxicillin
tibia, Rt. Inguinal LN enlarged and tendered                     B) oral amoxy-clavurunic acid
จะสงตรวจอะไรเพื่อวินิจฉัย                                       C) tympanoplasty

                         Exercise for National License Part II : March 2009 By NCTMS2009
                         หากมีขอเสนอแนะ ติชม เชิญแนะนําไดที่ www.nctms.in.th/webboard                  22
สหพันธนสิตนักศึกษาแพทยแหงประเทศไทย
                                                                                 ิ
23. เด็ก 9 เดือน พูดเปนคําๆ ไมมีความหมาย เกาะ                    A) meningitis
ยื น แต เ ดิ น ไม ไ ด ยอมให ทุ ก คนอุ ม เวลาขอของไม          B) hypoglycemia
ยอมให เด็กมีความผิดปกติดานใด                                     C) polycythemia
           A) speech                                               D) perinatal asphyxia
           B) walk                                                 E) intraventricular hemorrhage
           C) separation anxiety
           D) to give toy                                   27. เด็กอายุ 1 ป มีไข 1 วัน ซึมลง PE: BT 39.5°C,
                                                            BP 80/60 PR 140 RR 42, petechiae with
24. เด็กมีอาการปวดหัวมาก n/v ไข มีประวัติวาไป             purpura at lower extremities จงใหการรักษา
เลนน้ําในลําคลองมาดวย ตรวจรางกาย stiff neck              เบื้องตน
positive นาจะเกิดจากเชื้ออะไร                                        A) dextran
         A) Nae                                                       B) dopamine
         B) H. influ                                                  C) NSS
         C) Strep pneumo                                              D) platelet
                                                                      E) FFP
25. เด็ก 2 ป มาwell baby clinic เคยได BCG 1 เข็ม
HBV 3 เข็ม ( DDT,OPV 3 เข็ม ) JE 2 เข็ม MMR 1               28. เด็กชาย 3 ป มารพ.ดวยอาการปวดทอง
เข็ม ถามวาครั้งนี้ใหอะไร                                  PE : confuse, diaphoretic, profuse sweating,
         A) JE                                              drooling and miosis.
         B) JE, Hib                                         การรักษาที่ เหมาะสมที่สุดไดแก
         C) OPV, DPT,JE                                             A) ethanol
         D) OPV, DPT, Hib                                           B) atropine
                                                                    C) naloxone
26. ทารกแรกคลอด GA 41 wk BW 3400 g มี                               D) haloperidol
meconium stained AF APGAR= 2,5 ที่ 1 และ 5                          E) dimenhydramine
นาที 10 ชั่วโมงตอมามีอาการชักเกร็งทั้งตัว ตรวจ
เลือด glucose 50 Hct 58% คิดวาอาการชัก เกิด
จากอะไร




                        Exercise for National License Part II : March 2009 By NCTMS2009
                        หากมีขอเสนอแนะ ติชม เชิญแนะนําไดที่ www.nctms.in.th/webboard                     23
สหพันธนสิตนักศึกษาแพทยแหงประเทศไทย
                                                                                  ิ
29. เด็กอายุ 3 วัน อาเจียนน้ําสีเขียว กินนมแมตั้งแต       32. เด็กชายอายุ 1 ป มีไขมา 2 วัน ซีด ปสสาวะสีเขม
วันแรก มารดามี PROM 10 ชั่วโมงกอนคลอด ตรวจ                 กินยาลดไขกอนมารพ.
รางกายทองอืดเล็กนอย ไมซึม จะinvestigate อะไร             PE: moderate pale, mild icteric sclera,
        A) stool exam                                       tachycardia, no hepatosplenomegaly.
        B) CBC                                              CBC: Hct 24% WBC 11500 (N 76% L 21%
        C) H/C                                              M3%) plt. 470000
        D) plain film abdomen                               PBS: bite cell, basket cell, anemia
                                                            จงใหการวินิจฉัย
30. ในรพ.ชุมชน เด็ก 8 เดือนมาดวย clinical                          A) Hb H disease
meningitis ชัก ทําอยางไร                                           B) G 6 PD deficiency
      A) แจงพอแมกอนเพื่อ LP                                     C) PNH
      B) LP ไดเลยเพราะวาเปนเรืองเรงดวน
                                   ่                                D) AIHA
      C) สงรพ.จงหวัดใหเร็วที่สุด                                  E) hereditary spherocytosis
      D) CT brain
      E) เจาะเลือด H/C แลวให ATB                          33. เด็กหญิง 6 ป หกลมมีแผลที่หนาแขงขวา 1
                                                            สัปดาหกอน ทําแผลเองที่บาน 3 วันกอนเริ่มมีไข สูง
31. เด็ก 9 เดือน ยังไมเดน เกาะยืนได พูดเปนคําไมมี       แผลมี ห นองไหล 6 ชม.ก อ นมีเ จ็ บหน า อก เหนื่ อ ย
ความหมาย ยอมให ทุ ก คนอุ ม ฟ น ยั ง ไม ขึ้ น ไม ใ ห   หอบ บวม นอนราบไมได
ของเล น เวลาขอ เด็ ก คนนี้ มี พั ฒ นาการด า นใดผิ ด       PE : BT 39.8 BP 70/45 PR 140 RR 40
ปกติ                                                        no cyanosis, engorged neck vein, distant heart
         A) language                                        sound, crepitation both lungs, liver 3 cm BRCM
         B) walking                                         จงใหการรักษาเบื้องตน
         C) stranger anxiety                                        A) pericardiocentesis
         D) teeth impaction                                         B) dopamine
         E) don’t give toy when asked                               C) ET intubation
                                                                    D) CPR
                                                                    E) NSS




                        Exercise for National License Part II : March 2009 By NCTMS2009
                        หากมีขอเสนอแนะ ติชม เชิญแนะนําไดที่ www.nctms.in.th/webboard                       24
Exercise national license_part_ii_march_2009_2
Exercise national license_part_ii_march_2009_2
Exercise national license_part_ii_march_2009_2
Exercise national license_part_ii_march_2009_2
Exercise national license_part_ii_march_2009_2

Weitere ähnliche Inhalte

Ähnlich wie Exercise national license_part_ii_march_2009_2

Interhospital chest conference
Interhospital chest conferenceInterhospital chest conference
Interhospital chest conferenceMy Parents
 
Osce examination si116
Osce examination si116Osce examination si116
Osce examination si116vora kun
 
Compre si 2010 l
Compre si 2010 lCompre si 2010 l
Compre si 2010 lvora kun
 
Nle step 2_2009 si115-116 and nle_step_2_2009 nctms editors cut
Nle step 2_2009 si115-116 and nle_step_2_2009 nctms editors cutNle step 2_2009 si115-116 and nle_step_2_2009 nctms editors cut
Nle step 2_2009 si115-116 and nle_step_2_2009 nctms editors cutLoveis1able Khumpuangdee
 
กาย่า presentation
กาย่า presentationกาย่า presentation
กาย่า presentationCrystalpet
 
10แบบทดสอบภูมิคุ้มกันของร่างกาย (ตอนที่ 1)
10แบบทดสอบภูมิคุ้มกันของร่างกาย (ตอนที่ 1)10แบบทดสอบภูมิคุ้มกันของร่างกาย (ตอนที่ 1)
10แบบทดสอบภูมิคุ้มกันของร่างกาย (ตอนที่ 1)สำเร็จ นางสีคุณ
 
สอบสวนผู้ป่วยเสียชีวิตโรคไข้มาลาเรีย
สอบสวนผู้ป่วยเสียชีวิตโรคไข้มาลาเรียสอบสวนผู้ป่วยเสียชีวิตโรคไข้มาลาเรีย
สอบสวนผู้ป่วยเสียชีวิตโรคไข้มาลาเรียนายสามารถ เฮียงสุข
 
Trauma Initial assessment and Resuscitation
Trauma Initial assessment and ResuscitationTrauma Initial assessment and Resuscitation
Trauma Initial assessment and ResuscitationNarenthorn EMS Center
 
Mdcu Exam Step 2 2010
Mdcu Exam Step 2 2010Mdcu Exam Step 2 2010
Mdcu Exam Step 2 2010vora kun
 
Nl 2010 nctms
Nl 2010 nctmsNl 2010 nctms
Nl 2010 nctmsNew Srsn
 

Ähnlich wie Exercise national license_part_ii_march_2009_2 (16)

Interhospital chest conference
Interhospital chest conferenceInterhospital chest conference
Interhospital chest conference
 
National license 2010 by med tu 16
National license 2010 by med tu 16National license 2010 by med tu 16
National license 2010 by med tu 16
 
Osce examination si116
Osce examination si116Osce examination si116
Osce examination si116
 
Compre si 2010 l
Compre si 2010 lCompre si 2010 l
Compre si 2010 l
 
Nle step 2_2009 si115-116 and nle_step_2_2009 nctms editors cut
Nle step 2_2009 si115-116 and nle_step_2_2009 nctms editors cutNle step 2_2009 si115-116 and nle_step_2_2009 nctms editors cut
Nle step 2_2009 si115-116 and nle_step_2_2009 nctms editors cut
 
แบบฝึกหัด
แบบฝึกหัดแบบฝึกหัด
แบบฝึกหัด
 
test
testtest
test
 
กาย่า presentation
กาย่า presentationกาย่า presentation
กาย่า presentation
 
Lesson 1 homeostasis
Lesson 1 homeostasisLesson 1 homeostasis
Lesson 1 homeostasis
 
10แบบทดสอบภูมิคุ้มกันของร่างกาย (ตอนที่ 1)
10แบบทดสอบภูมิคุ้มกันของร่างกาย (ตอนที่ 1)10แบบทดสอบภูมิคุ้มกันของร่างกาย (ตอนที่ 1)
10แบบทดสอบภูมิคุ้มกันของร่างกาย (ตอนที่ 1)
 
เกมสี่ตัวเลือก
เกมสี่ตัวเลือกเกมสี่ตัวเลือก
เกมสี่ตัวเลือก
 
สอบสวนผู้ป่วยเสียชีวิตโรคไข้มาลาเรีย
สอบสวนผู้ป่วยเสียชีวิตโรคไข้มาลาเรียสอบสวนผู้ป่วยเสียชีวิตโรคไข้มาลาเรีย
สอบสวนผู้ป่วยเสียชีวิตโรคไข้มาลาเรีย
 
Rr rx
Rr rxRr rx
Rr rx
 
Trauma Initial assessment and Resuscitation
Trauma Initial assessment and ResuscitationTrauma Initial assessment and Resuscitation
Trauma Initial assessment and Resuscitation
 
Mdcu Exam Step 2 2010
Mdcu Exam Step 2 2010Mdcu Exam Step 2 2010
Mdcu Exam Step 2 2010
 
Nl 2010 nctms
Nl 2010 nctmsNl 2010 nctms
Nl 2010 nctms
 

Mehr von Loveis1able Khumpuangdee (20)

Rollup01
Rollup01Rollup01
Rollup01
 
Protec
ProtecProtec
Protec
 
Factsheet hfm
Factsheet hfmFactsheet hfm
Factsheet hfm
 
Factsheet
FactsheetFactsheet
Factsheet
 
Eidnotebook54
Eidnotebook54Eidnotebook54
Eidnotebook54
 
Data l3 148
Data l3 148Data l3 148
Data l3 148
 
Data l3 147
Data l3 147Data l3 147
Data l3 147
 
Data l3 127
Data l3 127Data l3 127
Data l3 127
 
Data l3 126
Data l3 126Data l3 126
Data l3 126
 
Data l3 113
Data l3 113Data l3 113
Data l3 113
 
Data l3 112
Data l3 112Data l3 112
Data l3 112
 
Data l3 92
Data l3 92Data l3 92
Data l3 92
 
Data l3 89
Data l3 89Data l3 89
Data l3 89
 
Data l2 80
Data l2 80Data l2 80
Data l2 80
 
Hfm reccomment10072555
Hfm reccomment10072555Hfm reccomment10072555
Hfm reccomment10072555
 
Hfm work2550
Hfm work2550Hfm work2550
Hfm work2550
 
Factsheet hfm
Factsheet hfmFactsheet hfm
Factsheet hfm
 
Publichealth
PublichealthPublichealth
Publichealth
 
แนวทางการดาเน ํ นงานป ิ องก ้ นควบค ั มการระบาดของโรคม ุ ือ เท้า ปาก สําหรบแพ...
แนวทางการดาเน ํ นงานป ิ องก ้ นควบค ั มการระบาดของโรคม ุ ือ เท้า ปาก สําหรบแพ...แนวทางการดาเน ํ นงานป ิ องก ้ นควบค ั มการระบาดของโรคม ุ ือ เท้า ปาก สําหรบแพ...
แนวทางการดาเน ํ นงานป ิ องก ้ นควบค ั มการระบาดของโรคม ุ ือ เท้า ปาก สําหรบแพ...
 
hand foot mouth
hand foot mouthhand foot mouth
hand foot mouth
 

Exercise national license_part_ii_march_2009_2

  • 1. สหพั น ธ นิ สิ ต นั ก ศึ ก ษาแพทย แ ห ง ประเทศไทย (สพท.) National Council of Thai Medical Students (NCTMS) วิทยาลัยแพทยศาสตรกรุงเทพมหานครและวชิรพยาบาล คณะแพทยศาสตร มหาวิทยาลัยขอนแกน คณะแพทยศาสตร จุฬาลงกรณมหาวิทยาลัย คณะแพทยศาสตร มหาวิทยาลัยเชียงใหม คณะแพทยศาสตร มหาวิทยาลัยธรรมศาสตร คณะแพทยศาสตร มหาวิทยาลัยนราธิวาสราชนครินทร คณะแพทยศาสตร มหาวิทยาลัยนเรศวร คณะแพทยศาสตร มหาวิทยาลัยบูรพา คณะแพทยศาสตรพระบรมราชชนก วิทยาลัยแพทยศาสตรพระมงกุฎเกลา คณะแพทยศาสตร มหาวิทยาลัยมหาสารคาม วิทยาลัยแพทยศาสตร มหาวิทยาลัยรังสิต คณะแพทยศาสตรโรงพยาบาลรามาธิบดี สํานักวิชาแพทยศาสตร มหาวิทยาลัยวลัยลักษณ คณะแพทยศาสตร มหาวิทยาลัยศรีนครินทรวิโรฒ คณะแพทยศาสตรศิรราชพยาบาล ิ คณะแพทยศาสตร มหาวิทยาลัยสงขลานครินทร สํานักวิชาแพทยศาสตร มหาวิทยาลัยเทคโนโลยีสุรนารี วิทยาลัยแพทยศาสตรและสาธารณสุข มหาวิทยาลัยอุบลราชธานี ๓๑๗/๕ ถนนราชวิถี แขวงทุงพญาไท เขตราชเทวี กรุงเทพมหานคร ๑๐๔๐๐ โทรสาร ๐-๒๓๕๔-๗๗๕๐ ตอ ๑๑๒ Exercise for National License Part II CLINICAL SCIENCE : March 2009 By NCTMS2009 แนวขอสอบชุดนี้เปนการรวบรวมแนวขอสอบที่สงมาจากโรงเรียนแพทย3แหง ไดแก (เรียงตามลําดับตัวอักษรภาษาอังกฤษ) CU คณะแพทยศาสตร จุฬาลงกรณมหาวิทยาลัย NU คณะแพทยศาสตร มหาวิทยาลัยนเรศวร PSU คณะแพทยศาสตร มหาวิทยาลัยสงขลานครินทร รวบรวมและเรียบเรียงโดย ฝายวิชาการ คณะแพทยศาสตร มหาวิทยาลัยนเรศวร ตรวจทานโดย คณะกรรมการสหพันธนิสิตนักศึกษาแพทยแหงประเทศไทย รุนที่19 หากมีขอผิดพลาดประการใด ขออภัยมาไว ณ ที่นี้ แนะนําหรือติชมไดที่ www.nctms.in.th/webboard 09/09/09
  • 2. สหพันธนสิตนักศึกษาแพทยแหงประเทศไทย ิ Medicine 4. ชาย 20ป ไขสูง5วัน ปวดหัว คลื่นไสแตไมอาเจียน 1.ชาย 18y เพื่อนนํามาสง ER เนื่องจากพบวานอน PE : mild conjunctival injection, no jaundice, สลบ ปลุกไมตื่น PE : GCS 4, Pupil 1 mm, RR 6 V/S : BT 39.5 C, PR 90, RR 14, BP 120/90 นอกจากชวยหายใจแลวจะรักษาอยางไรตอ liver 1 FB RBCM, span 11 cm, petechia on A) Naloxone extremities B) Atropine CBC : Hct 45% WBC 4,300 N50 L40 M3 C) Glucose atypL 7 plt 60,000 จงใหการวินิจฉัย 2. ชาย 20 ป ถูกงูกัด 1 ชม.ที่เทาเดินในสวน ตรวจ A) leptospirosis รางกายมี bilateral ptosis , Right foot 2 fnags B) DHF mark with swelling ถามวางูอะไรกัด C) scrub typhus A) งูเหา D) malaria B) งูกะปะ E) enteric fever C) งูแมวเซา D) งูทับสมิงคลา 5. ชาย 20 ป ไขสูง ปวดหัว E) งูเขียวหางไหม PE: subconjunctival hemorrhage BT 39 C, stiff neck positive, petechia and purpura on both 3. ถาม GCS ชายหลังประสบอุบัติเหตุ ไมลืมตา แต legs จงบอก organism เรียกแลวลืมตา บนพึมพํา พูดเปนคําไมรูเรื่อง ยก A) N. meningitides มือขึ้นมาปดได ถูกตําแหนงที่ทําใหเจ็บ B) K. influenzae A) 7 C) S. pneumoniae B) 8 D) S. aureus C) 9 E) M. pneumoniae D) 10 E) 11 6.ชายไข 5 วัน มีคลื่นไส ถายเหลว มีไอเจ็บคอรวม ดวย ตรวจรางกายตับโต Lab Pltลดลง Atyp lymph เพิ่มขึ้น A) DHF Exercise for National License Part II : March 2009 By NCTMS2009 หากมีขอเสนอแนะ ติชม เชิญแนะนําไดที่ www.nctms.in.th/webboard 1
  • 3. สหพันธนสิตนักศึกษาแพทยแหงประเทศไทย ิ 7.ชาย 30y ไข ป วดศี ร ษะมาก ซึ ม มี purpura, 12.ชายชาวนา บานอยูกาฬสินธุ เปนไข หอบเหนื่อย petichiae ขึ้นตามตัว stiffness positive เกิดจากเชื้อ CXR : Patchy infiltration Cr : raising ใดมากที่สุด U/S : multiple nodular hypoechoic mass A) Neisseria meningitides A) Melioidosis B) Stap aureus C) Haemophilus influenzae 13. หญิง 30 ป มีจุดเลือดออกตามตัวมา 3 สัปดาห บางครั้งมีเลือดออกตามไรฟนรวมดวย 8. Blood smear, พบ multiple ring form จะ รักษา PE: not pale, no jaundice, petechiae purpura at อยางไร trunk and extremities A) cholroquine CBC : Hb 12 Hct 36 MCV 80 WBC 4,600 N65 B) IV Artesunate+tetracycline L30 M5 Plt 36,000 C) chlroquine+primaquine จงหาการรักษา ที่เหมาะสม D) artesunate+mefloquine 5 quinine A) prednisolone B) cyclophosphamide 9.รูปไขพยาธิ Opistorchis viverrini Txยังไง C) IVIG A) Albendazole D) splenectomy B) Mebendazole E) plt transfusion C) Praziquantel 14.ชาย 30y คลําไดLN ที่คอโตทั้งสองขาง มา 3 10.ผูปวย Nephrotic syndrome on prednisolone เดือน กอนโตขึ้นเรื่อยๆ 1 สัปดาห รูสึกอึดอัดหายใจ นาน ถามวาเสี่ยงติด Parasite larva อะไร ลําบาก เสียงดังเวลาหายใจเขา A) Strongyloides PE : BP 140/90 RR 24 PR 100 inspiratory stridor, bilateral cervical LN 11.ผูปวยหญิงไปเที่ยวนราธิวาส เปนไขปวดขอ ผล enlargement 2 cm., lung clear with respiratory เลือดเปน Lymp เดน, Atyp 15% , Plt ต่ํา stridor , liver 3 cm. BRCM, spleen 8 cm. BLCM A) Lepto จงหาการรักษา เบื้องตน B) Scrub A) Beta2 agonist inhale C) Chicukunya fever B) Adrenaline SC C) Furosemide IV D) Dexamethasone IV E) ET tube Exercise for National License Part II : March 2009 By NCTMS2009 หากมีขอเสนอแนะ ติชม เชิญแนะนําไดที่ www.nctms.in.th/webboard 2
  • 4. สหพันธนสิตนักศึกษาแพทยแหงประเทศไทย ิ 15. ชายอายุ 50 ป สูบบุหรี่วันละ 1-2 ซองมา 30 ป 18.หญิง 70ป เปน Alzheimer 4-5ป 1ป กอนไมยอม สังเกตวามีหนาบวม หายใจแนนติดขัดมา 2 สัปดาห กินอาหาร ญาติใหอาหารดวยการบีบ syringe เขา PE : facial swelling, superficial vein dilatation at ปาก ไมมีอาการสําลัก แตผอมลง anterior chest wall, Chest –normal, Abd-normal, PE : generalize wasting, pale, pitting edema Other-WNL both legs, white nail จงใหการวินิจฉัย ทานจะเลือกใหอาหารผูปวยอยางไร A) Cor pulmonale A) Nasogastric tube B) Cardiac tamponade B) Nasoduodenal tube C) Nephrotic syndrome C) Nasojejunal tube D) Pulmonary TB D) Percutaneous endoscopic E) Bronchogenic CA gastrostomy tube E) Percutaneous endoscopic 16. ผูปวยชายอายุ 40 ป กินเหลามากเพลีย เปนมา jejunostomy tube 2 เดือน CBC : WBC 12,000 มี WBC granulocyte ทุก series , plt 650,000 splenomegaly จงใหการ 19.ชาย อายุ 40ป สวนสูง 160 cm. นน. 90 kg. วินิจฉัย FBS 140 mg%---repeat 145 mg% ไมมีอาการของ A) acute myelofibrosis DM ทานจะทําอยางไร B) acute myeloid leukemia A) สังเกตอาการ C) chronic myeloid leukemia B) ควบคุมอาหารและออกกําลังกาย D) malignant lymphoma C) Insulin injection D) กินยากลุม incretin  17.ชาย 35ป นน.137 kg. สูง 170 cm. รอบเอว 137 E) กินยาเพิ่มการหลัง insulin ่ cm.เปน HT 2y. BS 120 mg% สําหรับผูปวยรายนี้จะลดความอวนอยางไรใหไดผล 20. หญิง 48y TB 6/3 SGOT 50 SGPT 50 ALP 500 ระยะยาวที่สุด DB 6/3 U/S hyperecchoic no occupy lesion A) Life style modification เสี่ยงขาดสารใด B) Orlistat A) retinol C) Sibutramine B) zinc D) Gastric resection C) selenium E) GI bypass D) toliem E) colbalamine Exercise for National License Part II : March 2009 By NCTMS2009 หากมีขอเสนอแนะ ติชม เชิญแนะนําไดที่ www.nctms.in.th/webboard 3
  • 5. สหพันธนสิตนักศึกษาแพทยแหงประเทศไทย ิ 21. อาการ Parkinson รักษา 26. หญิง 40 ป เปนเบาหวาน น้ําตาล 134mg/dL A) Levodopa ปวดหูขางขวา บรรยายตรวจรางกายเหมือน Herpes B) Benzhexol zoster oticus มีกลามเนื้อใบหนาออนแรงแบบ Lower motor neuron จะทําอะไรตอ 22. เปน DM facial palsy ดาน? เหมือน LMN ( คิด A) CT scan วาเปน Bell’s palsy ) น้ําตาลไมสูง ถาม Tx B) ยากันชัก อะไรซักอยาง คนจดจําไมได A) Prednisolone C) Prednisolone D) ASA 23. ใหอาการมาประมาณ Myasthenia gravis คือ Proximal muscle weakness อาการเปนมากชวง 27. หญิง 30 ป มีอาการปวดหัวเปนๆ หาย ๆ 3-4 บายๆ – Proptosis bilateral ถามวาจะสงตรวจอะไร ครั้งตอเดือน เปนมา 2 ป ตรวจรางกายปกติดี เพื่อใหได Dx… A) diazepam A) Prostigmine test B) amitryptyline B) CT brain C) lorazepam C) Electrolyte D) CT brain D) Muscle biopsy 28.ชาย 24 ป ชัก 1 hr ชักตั้งแตอายุ 15 ป ชัก 3-4 24. PE พบชาที่นิ้วโปงกลางชี้ , Tinel – negative จะ ครั้ง/ป ไมไดรักษามา 1 ป ตอนนี้ไมมีอาการ สงตรวจอะไร A) diazepam A) Cholesterol B) phenyltoin B) LDL C) phenobarb C) Cervical traction D) midazolam D) Pronator … E) gabapentin 25 คนไข 75 ป ใหอาการมาแบบ Parkinson เลย 29. ชาย 30 ป มาดวยชาตั้งแตหัวไหลไปถึงปลายเทา PE:Cocwheel rigidity , resting tremor ตรวจร า งกายพบ กล า มเนื้ อ กางแขน อ อ นแรง ใหยา อะไร กลามเนื้อขากระตุกเกร็ง lesion อยูที่ใด A) Levodopa A) cerebral cortex B) cervical cord C) cervical nerve root Exercise for National License Part II : March 2009 By NCTMS2009 หากมีขอเสนอแนะ ติชม เชิญแนะนําไดที่ www.nctms.in.th/webboard 4
  • 6. สหพันธนสิตนักศึกษาแพทยแหงประเทศไทย ิ 30. ชาย 18 ป MCA สมองบวม ใส ET tube มา 2 34. ชาย 30 ป เหนื่อยมา 2 hr. สูบบุหรี่ครึ่งซอง/วัน เดือน หลังoff tube มีอาการหายใจขัด มี stridor ทั้ง มาเป น เวลา15 ป มี อ าการไอ หอบ หอบเวลา เขาและออก เวลาพูดมีลมออกรั่วออกมา กลางคืน+เวลาอากาศเย็น จงใหการวินิจฉัย จง diagnosis A) Bronchiectasis A) vocal cord granulation B) Bronchial asthma B) arythnoid dislocation C) Chronic bronchitis D) Pulmonary TB 31 ชาย 33ป ปวดหัว มาหลายเดือน ใจสั่น เหนื่อย E) CA bronchus งาย flushing BP 180/110 PR 110 investigation? A) urine VMA 35. ชายไขต่ําๆ ไอ หอบ มา 2 วัน ฟงปอดมี B) CT brain crepitation ดานขวา CXR:RLL infiltration ถามเชื้อ C) Thyroid function A) H.influenza D) pyelogram B) S.pneumonae C) Mycloplasma pnemoniae 32. ผูปวยหญิง อายุ 44 ป สูง 155 cm. BW: 77 kg. D) Chlamydia ชอบกินอาหารรสจัด PE: BP สูง PR: 78/min E) S.aureus จงmanagement A) ACEI 36. หญิง 28 ป ไขต่ําๆ ไอ มีเสมหะ มี bullous B) B-blocker myringitis คิดถึงเชื้ออะไร C) CCB A) S.pneumoniae D) diuretic B) H.influenzae E) life style modification C) RSV D) Mycoplasma 33. เหมือนโพย NT 2007 ผูปวยกินเหลา มี HF จงmanagement A) Digoxin B) Thiamine Exercise for National License Part II : March 2009 By NCTMS2009 หากมีขอเสนอแนะ ติชม เชิญแนะนําไดที่ www.nctms.in.th/webboard 5
  • 7. สหพันธนสิตนักศึกษาแพทยแหงประเทศไทย ิ 37. หญิงไมมีอายุ (โจทยไมใหมา) คันที่ขาซายมาก A) serum Ig A level PE พบ ill-defined erythematous papulovesicle B) serum C 3 complement confluence to plaque at left leg ใหยาอะไรดี C) ANA A) antibiotic D) ASO titer B) antiviral E) serum cholesterol an albumin C) antifungal D) antihistamine 41.หญิง70ป มีไข ปวดเมื่อยตัว1wk กอนหายเอง PE E) corticosteroid : multiple erythematous subcutaneous nodusum ควรสงตรวจอะไร 38. คนไขมีผื่นเปนตุมน้ําตามตัว แขนขา A) CXR PE: discrete vesicle, bullae of trunk, low B) UA extrimities การตรวจใดชวยวินิจฉัย C) Plain abdomen A) gram stain D) ESR B) AFS C) Tzack smear 42.ชายพบผื่นที่ขาหนีบลามชาๆ well define scaly annular with arciform erythematous patch สง 39. หญิง 29 ป เปนไข ปวดเมื่อยกลามเนื้อ ไมไดกิน KOH จะพบอะไร ยา หายเองได ตอมามี subcutaneous nodule at A) Short hyphae with thick oval leg สงตรวจทางหองปฎิบัติการใด B) Pseudohyphae A) ESR C) Branching hyphae with spore B) x ray D) Septated hyphae with arthrospore C) CBC 43. ผูปวยชายปวดขอเขาขวา มีไข เจาะหนองที่เขา 40. เด็กผูหญิง 15 ป บวมทั้งตัวมา 1 เดือน ปวดขอ ได 1 ml ยอม gram ได gram+ cocci in cluster เขา ขอเทา มา มี oral ulcer ที่ upper palate 2 Tx ดวย Cloxacillin ไมดีขึ้น เขาขวาบวมมากขึ้น สัปดาห ตอมา เหนื่อยมากขึ้น, V/S : BT 37.8°C BP ควรทําอะไรตอ 140/90 mmHg RR 24 /min, erythematous A) ให NSAIDs maculopapular rash both palm an soles, ESR B) ให Ceftriaxone 100 mm/hr, Lab: UA :protein 4+ ,RBC C) เปลี่ยนยาเปน Vancomycin numerous , granular,RBC cast D) ใหยาเดิม แตเจาะเขา (aspirate) หนอง สงวาสงตรวจอะไรที่ชวยวินจฉัยมากที่สุด ิ ออกใหมากที่สุด Exercise for National License Part II : March 2009 By NCTMS2009 หากมีขอเสนอแนะ ติชม เชิญแนะนําไดที่ www.nctms.in.th/webboard 6
  • 8. สหพันธนสิตนักศึกษาแพทยแหงประเทศไทย ิ 44. หญิง 43y มีอาการออนเพลียงาย กลามเนื้อแขน 48. ชายอายุ 45 ป ภูมิลําเนา อ.สังขละบุรี ขาออนแรง เคยมีอาการบวมที่ขา กินHCTZ 50 mg จ.กาญจนบุรี มีอาการเพลีย ออนแรงมา 3-4 เดือน มาตลอด อาการในผูปวยรายนี้นาจะเกิดจากอะไร PE : generalized hyperpigmentation, A) hyponatrimia hyperkeratosis at palms and soles, B) hypokalemia whiteline at finger nails C) hypomagnesia Motor power – proximal III/V, distal IV/V D) hypocalcemia Decreased pain and touch sensation จะใหการรักษาอยางไร 45.ใหรูปงูม าลัก ษณะเป น สี ดํา สลั บ เหลื อ งเข ม ลาย A) Dimercaptol แบบสามเหลี่ ย ม (เป น งู ก ะปะครั บ ) ถามว า จะ B) Calcium EDTA investigate อะไร C) Penicillamine A) VCT D) Deferoxamine B) Pulmonary function test E) Dialysis C) Peak flow 49. ผูปวยหมดสติ พูดออแอ ไมเปนคํา เรียกแลวลืม 46. รูปงูบนใบตอง ( คนไขนํางูมาดวย) สงตรวจอะไร ตาตื่ น ดี กระตุ น ให เ จ็ บ แล ว ป ด ตํ า แหน ง ที่ เ จ็ บ ได มี ในผูปวยรายนี้ GCS เทาไหร A) peak flow A) 7 B) VCT B) 8 C) BUN/Cr C) 9 D) UA D) 10 E) 11 47. ชายอายุ 40 ป ทําอาชีพปลูกผัก ไขมา 5วัน ปวด ศีรษะมาก มี LN cervical โต 2 ขาง conjunctival 50.นางสาวก.ตองการสมัครเปนพยาบาลจึงมาตรวจ injection, no jaundice, stiffneck negative CBC : สุขภาพ ผลปรากฎวา HBsAg +, HBsAb – ผล liver WBC 10,000 PMN 50% Lymph 50% จงใหการ function test ปกติ ควรทําอยางไรมากที่สุด วินิจฉัย A) รักษาดวย interferon A) melioidosis B) ให HBV vaccine B) scrub typhus C) ตรวจ US abdomen C) dengue infection D) ตรวจ AFP D) viral meningitis E) ตรวจ HBsAg ซ้ําในอีก 6 เดือน Exercise for National License Part II : March 2009 By NCTMS2009 หากมีขอเสนอแนะ ติชม เชิญแนะนําไดที่ www.nctms.in.th/webboard 7
  • 9. สหพันธนสิตนักศึกษาแพทยแหงประเทศไทย ิ 51. ชายอายุ 30 ป คลํากอนไดที่คอโตมา 3 เดือน 54. ผูปวยหญิง ซีดมา 2 วัน reticulocyte count 8% กอนโตขึ้นเรื่อยๆ 1 สัปดาหกอนรูสึกอึดอัด หายใจ MCV 65 Hct 20% จงใหการวินิจฉัย ลําบาก มีเสียงดังตอนหายใจเขา PE: BP 140/90 A) thalassemia disease PR 100 RR 24, inspiratory stridor, lung B) AIHA clear, matted bilateral cervical neck C) G 6 PD deficiency cnlargement 4-8 cm in diameter จงบอก initial D) HS management A) Beta 2 agonist inhaler 55. ชายอายุ 40 ป บานอยูจังหวัดนราธิวาส มีไขมา B) SC adrenaline 4 วัน ตอมามีผื่นขึ้น 2 วันตอมาเริ่มมีอาการปวดเขา C) IV furosemide 2 ขาง วันนี้มีอาการปวดขอเทาและขอศอกทั้ง 2 ขาง D) IV dexamethasone ตรวจรางกาย BT 40C BP 100/80 E) endotracheal intubation maculopapular rash at trunk and extremities, swelling warm and tenderness with limitation of 52. ชายอายุ 25 ป มาดวยไข ปวดศีรษะ ชักเกร็งที่ movement both knees, ankles and elbows. Hct หนาและแขนดานซาย LP WBC 12 (L 95%) 40% WBC 3500 (N 45% L50% AL 5%) plt. protein 65 glucose 45 RBC 10-20 gram stain 45000 โรคใดที่ผูปวยมีโอกาสเปนมากที่สุด and india ink negative จงใหการรักษา A) ricketsial infection A) acyclovir B) Chigunkunya infection B) antifungal C) acute gouty arthritis C) antibiotics D) acute rheumatoid arthritis D) antiTB E) disseminated gonococcal arthritis 53. ผูปวย on high dose steroid ไอมีเลือดปน ตรวจ 56. ผูปวยหญิง มี trauma มา แลวมีอาการปวดขอ sputum เปนดังรูป นาจะเปนตัวอะไร เขาได ATB cloxacillin ประมาณวาเปน septic A) strongiloid knee 2 วันแลวอาการไมดีขี้น จง management B) ascaris A) เปลี่ยน ATB C) paragonimus B) ATB เดิม แลวทํา drainage ใหมากทีสุด ่ D) enterobius C) เพิ่ม dose ATB E) necator Exercise for National License Part II : March 2009 By NCTMS2009 หากมีขอเสนอแนะ ติชม เชิญแนะนําไดที่ www.nctms.in.th/webboard 8
  • 10. สหพันธนสิตนักศึกษาแพทยแหงประเทศไทย ิ 57. หญิงอายุ 60 ป บานอยูจังหวัดกาฬสินธุ มีไข ไอ 60. ผูปวยชายถูกงูกัดบริเวณขอเทาซาย เรื้อรังมา 1 เดือน เหนื่อยงายขึ้นมา 2 สัปดาห PE: ptosis both eyes, swelling at left ankle, fang PE : BT 39C RR 28 PR 100 BP 140/80 mark เกิดจากงูชนิดใด pale conjunctiva, no icteric sclerae, liver just A) งูเหา palpable, positive splenic dullness. B) งูเขียวหางไหม LAB: glucose 260 BUN 50 Cr 4.0 C) งูกะปะ CXR: patchy infiltration both lower lung field. D) งูทับสมิงคลา US abdomen: splenomegaly with multiple E) งูทะเล hypoechoic lesions ผูปวยนาจะเปนโรคใดมากที่สุด 61. ผูปวยชายไทยอายุ 55 ป กินเหลามานานและ A) Leptospirosis ตรวจพบว า เป น ตั บ แข็ ง มา 4 ป มี อ าการปวดแน น B) Melioidosis ทอ งมา 2 สั ป ดาห ปวดบริ เ วณใต ลิ้ น ป แ ละใต ช าย C) Salmonellosis โครงขวา ปวดตลอดเวลา ไมสัมพันธกับมื้ออาหาร D) disseminated TB USG: solid ill-defined mass 7 cm. in diameter at E) amoebic liver abscess right hepatic lobe การสงตรวจในขอใดชวยในการ วินิจฉัยไดดีที่สุด 58. ผูปวยชาย underlying cirrhosis มาดวยอาเจียน A) CEA เปนเลือด ตรวจรางกายพบ marked ascites, B) CA 19-9 crepitation both lungs C) β-HCG Hct 35% plt. 60000 APTT,PT prolonged D) α-fetoprotein ควรให blood product ตัวใด E) alkaline phosphatase A) PRC B) plt. conc. 62. ผูปวยหญิงอายุ 18ป 3 วันกอนมีผื่นขึ้นที่ขาซาย C) FFP เปน ill-defined erythematous papulovesicular D) cryoprecipitate confined to plaques ควรใช topical agent ตัวใด E) cryo-removed plasma A) ATB B) Antifungus 59. หนอไมดอง และเกิดอาการออนแรง เกิดจากสาร C) Antivirus ใด D) Antiparasite A) Botulinum toxin E) Steroid Exercise for National License Part II : March 2009 By NCTMS2009 หากมีขอเสนอแนะ ติชม เชิญแนะนําไดที่ www.nctms.in.th/webboard 9
  • 11. สหพันธนสิตนักศึกษาแพทยแหงประเทศไทย ิ 63. ผูปวยหญิงมาดวยอาการเหนื่อยหอบ Surgery CXR : patchy infiltration, 1.หญิง 30 ป มาดวยกอนที่เตานมขวา PE : cystic Sputum exam : pleomorphic gram positive mass 2 cm at right breast, not tender at lump, cocci. ไดยา Augmentin มา 3 วันอาการไมดีขึ้น no axillary lymphadenopathy investigation ? CXRซ้ํา พบpleural fluid A) incision ผลfluid analysis: wbc 2500 N90% LDH 900 B) excision จง ใหการรักษา C) FNA A) เพิ่ม aminoglycosides D) Reexamination next 6 months B) เปลียนเปน carbapenem E) Mammogram C) ใส ICD และให ATB เดิม D) Observe 2.ชาย 55y ประวัติดื่มสุราเปนประจําและเปนโรคตับ แข็งนาน 4y มีอาการแนนทองใตชายโครงขวาและลิน ้ 64. ชายอายุ 20 ป มาดวยปสสาวะสีเขมมา 2 วัน BP ป นาน 2y ตรวจ U/S Hyperdensity ,ill define 140/100 UA RBC>100/hpf WBC 10/hpf with red mass,เสนผานศูนยกลาง 7 cm.จะสง lab ใดเพื่อ cell cast protein 1+ ควรรักษาดวยยาใด ชวยในการวินิจฉัย. A) Norfloxacin A) ALP B) Furosemide B) AFP C) Prednisolone C) Beta-hCG D) Cef-3 D) CEA E) Sodium nitroprusside E) CA19-9 65. หญิงอายุ 70 ป tense bullae with vesicle on 3. หญิง 38 ป มีไข ปวดทอง ตรงสะดือ 4 ชั่วโมง trunk no mucosal invole ควรตรวจทดสอบดวย คลื่นไส อาเจียน เบื่ออาหาร ยายไปที่RLQ อะไร PE: mild tenderness RLQ A) Gram stain A) acute appendicitis B) KOH B) acute diverticulitis C) AFB C) ovarian twist D) Tzank smear D) pancreatitis E) india ink E) pelvic inflammation Exercise for National License Part II : March 2009 By NCTMS2009 หากมีขอเสนอแนะ ติชม เชิญแนะนําไดที่ www.nctms.in.th/webboard 10
  • 12. สหพันธนสิตนักศึกษาแพทยแหงประเทศไทย ิ 4. ชายอายุ 40 ป มาดวยปวดทองบีบ ๆ มีคลื่นไส ไม 7.ชายอายุ 60ป มีไข ตาเหลือง ตัวเหลือง มี อ าเจี ย น ยั ง ถ า ยอุ จ จาระได ป กติ เคยผ า ตั ด PE: liver กดเจ็บโต 3cm. below costal margin appendectomy เมื่อ 10 ปกอน U/S hypoechoic mass 6 cm.at segment 6 with PE: mild dehydration, moderate abdominal intraseptate, multiple gallstone and CBD stone, distention, no guarding, hyperactive bowel mild dilate of intrahepatic duct and common sound, PR : yellow stool การสงตรวจขอใด bile duct เหมาะสมที่สุด A) Hepatoma A) gastroduodenostomy B) Infect hepatic cyst B) ultrasound C) Cholangio CA C) RBC scan D) Amebic liver abscess D) plain film abd. supine and upright E) Pyogenic liver abscess E) barium enema note: ควรอานเรื่องแนวทางการ management gut 8. ชายโดนแทงหนาอกซายใตราวนม RR 24 PR 98 obstruction BP 112/60, decreased breath sound Lt. lung, trachea in midline ใส ICD แลวมีลมออกมา หลัง 5. ชาย 30 ป ผาตัด appendectomy 3 วันตอมา มี จากทํา CXR พบวามี pneumothorax 30% จะทํา ไข ปวดทอง อยางไรตอไป PE mild tenderness, แผลแดง มี fluctuation A) observe อยางใกลชิด จะทําอยางไรตอไป B) revise ICD A) ตัดไหม C) set OR emergency for thoracotomy B) ตัดไหม กิน ATB D) ใส ICD เพิมอีก 1 เสน ่ C) ตัดไหม แหวกแผล E) เช็คระบบ ICD D) กิน ATB E) ทา ATB 9. ผูปวยใสทอชวยหายใจ ผาตัด thyroid postop แพทยไปเยี่ยมผูปวยพบหายใจดี แตเมื่อพูดมีเสียง 6. แมบาน รูสึกชานิ้วโปง ชี้ กลาง บางครั้งเจ็บจนตื่น ออกมาดวย จงใหการวินิจฉัย กลางดึก ควรรักษาอยางไร A) Vocal cord paralysis A) surgery B) Vocal cord edema B) steroid injection C) Arrythenoid dislocation C) cervical traction D) Vocal cord granuloma Exercise for National License Part II : March 2009 By NCTMS2009 หากมีขอเสนอแนะ ติชม เชิญแนะนําไดที่ www.nctms.in.th/webboard 11
  • 13. สหพันธนสิตนักศึกษาแพทยแหงประเทศไทย ิ 10. ผูปวยหญิงกินยาลางหองน้ํา ถาม complication A) ไมตองทําอะไร ระยะยาวคืออะไร B) ให rabies vaccine ที่ day 0 A) Esophageal stricture C) ให rabies vaccine ที่ day 0,3 B) Mucolitis(มั้ง) D) ให rabies vaccine ที่ day 0,3,7 C) Esophagitis E) ให rabies vaccine ที่ day 0,3,7,14,28 D) อรอยติดใจ...กินเอง Ortho 11. ผูปวย trauma PE: BP drop , neck vein 1. ผูปวยชายอายุ 25 ป ขับรถจักรยานยนตมา 1 engorge, trachea shift to the left , decrease ชั่วโมง ตรวจรางกายพบ 10 cm. laceration wound breath sound ขอใดที่จําเปนตองรีบใส ICD มาก at right leg with tibial fracment exposure ทําการ ที่สุด ลางแผลและ debridement หลังจากนั้นจะทําการ A) neck vein engorge รักษาอยางไร B) trachea shift, decrease breath sound A) casting C) BP drop B) external fixation C) internal fixation 12. ผูชายอายุ 40 ป มีหนองไหลที่ขางรูกนมา 3 D) suture wound เดือน ไมไดรักษา ตอมาหนองแตกเอง กินยา ATB E) skeletal traction เอง จากนั้นอีก 3 เดือน บริเวณรอบกนมีตุมขนาด 0.5 cm ที่บริเวณเดิม มีรูเปด มีหนองและเลือดไหล 2. ผูชายอาชีพชางไม complain เรื่องปวดศอกขวา ออกเปนครั้งคราว กินยาแลวไมดีขึ้น จงวินิจฉัย ตรวจรางกาย tender at Rt. lateral epicondyle of A) hydradenitis suppurativa humerus ควรทํา test ใดเพื่อการวินิจฉัย B) anal fissure A) Tinel test at forarm C) anorectal abscess B) Phalan test D) herpes simplex C) resist wrist extensor test E) fistula in ano D) resist pronation 13. เด็กหญิงอายุ 6 ปถูกสุนัขกัดที่เทาเปนแผลยาว 2 ซม.เคยไดรับวัคซีนพิษสุนัขบาครบเมื่อ 1 ปกอน ตอง ใหการดูแลรักษาอยางไร Exercise for National License Part II : March 2009 By NCTMS2009 หากมีขอเสนอแนะ ติชม เชิญแนะนําไดที่ www.nctms.in.th/webboard 12
  • 14. สหพันธนสิตนักศึกษาแพทยแหงประเทศไทย ิ 3. ผูปวยหญิงอายุ 50 ป ตกจากที่สูง มีแขนผิดรูป มี 5. ผูชาย เลนฟุตบอล ลม เจ็บหัวไหล พบ neck of อาการปวดแขน บวมกระดกขอมือไมได humerus fx กลามเนื้อใดจะเสีย PE: deformity of arms with crepitus , A) Triceps dorsiflex grade: 0 , B) Biceps finger extension grade : 0 , C) Deltoid decrease sensation at dorsum at 1st web D) Coracobrachialis space of hand , E) Brachilis Film : oblique middle 1/3 fracture of right Humerus 6. ชาย อายุ 63ป มีอาการปวดสะโพกราวลงขา ปวด จง management มากขึ้นเมื่อเดินไดระยะ 300 m. ตองพักประมาณ 5 A) ใสเฝอกแขนทอนบนและทอนลาง นาที อาการดีขึ้นจึงเดินตอไดมา 6 เดือน 1 เดือนตอ B) ทํา EMG และใสเฝอกแขนทอนบนและ อาการเปนมากขึ้น เดินได 100 m. PE : Loss lumbar ทอนลาง lordotic มี tender at lumbar จากการ Dx คิดถึงโรค C) ผาตัดดามกระดูกและตอเสนประสาท ใดมากที่สุด D) ผาตัดดามกระดูกและใสเฝอกทอนบน A) Spinal instability และทอนลาง B) Spinal stenosis E) ผาตัดตอเสนประสาทและใสเฝอกแขน C) Spondylosis ทอนบน D) Chronic arterial occlusion E) Protuse nucleus proposus 4. ชายปวดขอศอกขวาดาน lateral epicondyle of humerus จะตรวจรางกายอะไรเพื่อยืนยันการDx 7. ชายอายุ 17 ป ใหประวัติวา 1 สัปดาหไดรับ A) Yegason’s test อุบัติเหตุที่เขาขวาจากการเลนฟุตบอล 4 วันกอนมา B) Valgus testของขอศอกขวา รพ. มีไขและเขาบวม เมื่อวานปวดเขามากจนเดินกระ C) Tinel testของทั้งแขนขวา เผลก จะสง investigation อะไรที่เหมาะสมที่สุด D) ทํา hyperextension of Rt.wrist A) ASO titer E) ตรวจ sensory ที่ first web space B) hemoculture C) X-rays D) arthrocentesis E) uric acid analysis Exercise for National License Part II : March 2009 By NCTMS2009 หากมีขอเสนอแนะ ติชม เชิญแนะนําไดที่ www.nctms.in.th/webboard 13
  • 15. สหพันธนสิตนักศึกษาแพทยแหงประเทศไทย ิ 8. หญิงอายุ 80 ป หกลมในหองน้ํา film พบ 12. ผูหญิง 63 ป ปวดหลังราวลงขา 2 ขางมา 6 intertrochanteric fracture จะทําอย า งไร เดือน เดินไดประมาณ 300 เมตรจะเริ่มมีอาการ ตอง A) surgery นั่ง ยองๆพัก อาการจะดีขึ้น อาการแยลงเรื่อยๆ เดิน B) long leg cast ไดเหลือ 100 เมตรก็ตองหยุดพัก ตรวจรางกาย C) ใหนงรถเข็น ั่ loss of lumbar lordosis, otherwise normal D) traction จง ใหการวินิจฉัย E) ประเมินสภาพผูปวยกอนเลือกวิธีการ  A) lumbar spine instability รักษา B) lumbar stenosis C) spondylolisthesis 9. ผูปวยชาย ปวดแขนและขอมือขวา ตรวจรางกาย D) arterial insufficiency มี weak abductor pollics brevis มีอาการชาที่ 3 นิ้วครึ่งดาน radial side ทํา Tinel test negative at 13. ผูปวยชาย ถูกรถชนที่หนาแขง ตรวจรางกายมี wrist ควรทํา test อะไรตอ แผลขนาด 1cm คลํา dorsalis pedis และposterior A) resist wrist extension test tibial pulse ไมได Doppler study ไม พบ flow B) resist wrist flexion test X-ray พบ comminuted fracture and posterior C) resist pronation test displacement of tibia ถามวาเปน open fracture D) resist supination test class ใด A) class I 10. What is the most common of low back pain? B) class II A) spondylosis C) class IIIa B) spondylolysis D) class IIIb C) loss of lumbar lordosis E) class IIIc D) spinal instability E) weak back muscle Rehab 1. หญิงปวดหลัง 5 เดือน ราวไปขา SLRT negative, 11. แมบานมีอาการปวดแขนขวา ขอมือขวา อาการ lost sense S1 นอกนั้นปกติ ทําอะไร เป น มากขึ้ น ตอนกลางคื น จนสะดุ ง ตื่ น มี อ าการชา A) ใสรองเทาเสริมสน ที่นิ้วโปง นิ้วชี้ นิ้วกลาง การรักษาที่เหมาะสมคืออะไร B) abdominal strengthening exercise A) cervical traction C) back strengthening exercise B) steroid injection C) warm pack Exercise for National License Part II : March 2009 By NCTMS2009 หากมีขอเสนอแนะ ติชม เชิญแนะนําไดที่ www.nctms.in.th/webboard 14
  • 16. สหพันธนสิตนักศึกษาแพทยแหงประเทศไทย ิ Eye ENT 1. มีอาการปวดตาทันที ประมาณวา AACG มี 1. ผูปวยบานอยูใกลสนามบิน มาดวยหูอื้อ ตรวจ ประวัติโรค asthma แพ sulfa management อยางไร Weber test : no lateralized , Renne test AC>BC A) atropine , Audio : เสียที่ 4000Hz จงใหการวินิจฉัย B) b-block A) Acoustic trauma C) alpha 2 agonist B) Meneire D) acetazolamide C) Atosclerosis 2. ผูหญิง 58 y เปน DM ตาขวามัวลง เห็นเหมือน 2. ชายไทย ขึ้นลง นั่งเครื่องบิน มีอาการปวดหู อะไรลอยไปลอยมา PE : Rt retinal hemorrhage , PE : hemotympanum จะรักษาอยางไร exuate , cotton wool spot, neovascularization at A) Decongestant disc จะmanage ยังไง B) Tympanectomy A) laser 3. คนงานโรงงานเหล็กเสน มีอาการหูเสื่อมจากเสียง 3. complication ของ traumatic hyphema คืออะไร ดัง การรักษาที่ตองทําทุกราย คือ A) strabismus A) ใหคนหูเสื่อมออกจากงาน B) glaucoma B) ใสเครื่องปองกันเสียงขณะทํางาน C) ambyopia C) ยายไปแผนกที่ไมมีเสียงดัง D) conjunctivitid 4. ผูปวยมาดวยอาการหูอื้อขางขวา 4. ผูปวยชายไมมีโรคประจําตัว มาดวยอาการตาแดง Weber: lateralization to the left ไดประวัตวาไปวายน้ําเมื่อวันกอน เชื้อใดนาจะ เปน Rinne: AC>BC both ears สาเหตุมากที่สุด จงใหการวินิจฉัย A) Naegeria foweri A) Otoscleriosis B) Pseudomonas B) Tympanoscleriosis C) H. influenza C) Otitis media wit effusion D) S. aureus D) Osiicular chain disruption E) SNHL Exercise for National License Part II : March 2009 By NCTMS2009 หากมีขอเสนอแนะ ติชม เชิญแนะนําไดที่ www.nctms.in.th/webboard 15
  • 17. สหพันธนสิตนักศึกษาแพทยแหงประเทศไทย ิ 5.ผูปวยเด็กมาดวยอาการปวดหู หลังเปนหวัด รักษา Radio ac otitis media ดวย Amoxicillin แลวไม หายใน 2 1. Sphenoid sinus เห็นไดชัดในภาพถายทาใด วัน ควรรักษาอยางไร A) lateral view A) Switch to Augmentin B) Towne view B) Switch to Caebapenenm C) straight PA view C) Add Aminoglycosides D) Inclined PA view D) Myringotomy E) Water view 6. ผูปวยมาดวยปวดหู ขยับหูไดลดลง มีอาการหนา OB & Gyn เบี้ยวขางเดียวกัน ตรวจพบ external auditory canal 1.หญิง25y GA 39wk เจ็บครรภคลอด บวม มี non mucous plug with granulation tissue PE : FH ¾ เหนือ umbilicus จงใหการวินิจฉัย PV : Cx 3cm. eff 50% ตรวจคลําพบคางเด็กอยู A) Cholesteatoma ดานหลังซายของมารดา ถาม fetal position B) Malignant otitis externa A) right mento posterior B) left mento posterior 7. ผูปวยชายอายุ 18 ป เกิดอุบัติเหตุจําเปนตองใส C) right occiput posterior ET tube นาน 10 วัน จึง off ETT 2 weeks ตอมา D) left occiput posterior ผู ป ว ยมาด ว ยอาการเหนื่ อ ยง า ย ตรวจร า งกายพบ E) mento posterior bilateral stridor จงใหการวินิจฉัย A) Subglottic stenosis 2. หญิง GA (term แลว ) เจ็บครรภคลอด ไดรับการ B) Unilateral vocal cord paralysis drip oxytocin ตอมาปวดทอง พบรอยคอด 2/3 เหนือ C) Intubation granuloma หัวเหนา management? A) emergency c/s 8.หลังผา thyroid ผูปวยเกิดอาการเสียงแหบ พูด B) oxytocin dosage reduction แลวมีเสียงลมออกมา จงใหการวินิจฉัย A) Subglottic stenosis 3. ถาม dose postcoital pill หลังมี SI 2 hr B) Unilateral vocal cord paralysis A) estradiol 60mg, next 60mg in the next 12 hr C) Intubation granuloma B) estradiol 100mg,next 100mg in the next 12hr C) estradiol 100mg, next 60mg in the next 12hr Exercise for National License Part II : March 2009 By NCTMS2009 หากมีขอเสนอแนะ ติชม เชิญแนะนําไดที่ www.nctms.in.th/webboard 16
  • 18. สหพันธนสิตนักศึกษาแพทยแหงประเทศไทย ิ 4.ผูปวยหลังคลอด มี Bleed 1500 cc. 8.หญิงทองแรก GA 40 week cervix เปด .. cm PE : Ut. Floppy , มีเลือดบริเวณ Vg effacement 50 % station -1 ischial spine หางกัน จง Dx 11 cm , intertuberous difference 8 cm ตอมาอีก A) Ut atony 2 hr ตรวจภายในยังเหมือนเดิม สาเหตุของการติดขัด B) Coagulopathy ของการคลอด คือ C) Bleed from varicose vein at Vg A) pelvic inlet D) Vg hematoma B) midplane E) Retain placenta C) pelvic outlet D) inlet+mid 5. หญิง G1P0 GA 8 wkมีเลือดออทาง Vg 2 วัน มีใจ E) mid+outlet สั่น อาเจียนมาก คลื่นไส BP 140/90 PR 110 RR 16 คลําไดยอด Ut สูงกวา suprapubic 9. หญิง 30 ป GA 34 week มีอาการปสสาวะลําบาก A) Thyrotoxicosis และแสบรอนบริเวณชองคลอด B) Throphoblastic PE : vesicle& shallow ulcer at valva C) Twin จงใหการวินิจฉัย D) Preeclampsia A) syphilis E) Preg with myomauterie B) herpetic vulvitis C) Lipsmutz 6. หญิง Preg G2 มี second half bleeding มี D) molluscum contagiosum เลือดออกทาง vagina เปน painless bleeding จะ ทําอะไรตอ 10.ชาย 40ป หญิง 38ป มาปรึกษาแพทยดวยเรื่องลูก A) ultrasound คนกอนมีพัฒนาการชา ควรทําอยางไร A) ตรวจ chromosome 7. หญิง Preg มาฝากครรภ ตรวจ Hb เปน AE สามี เปน AA2 (A2:3%, F: 1%) จะmanagementอยางไร 11. หญิง 20 ป ปวดทองนอย มีเพศสัมพันธครั้งแรก A) U/S เมื่อ 1 วันกอน UA:WBC 25cell , RBC 3-5cell B) ทํา amniocentesis กลไกการเกิดโรค C) ทํา cudocentesis A) Ascending urethral transmission D) ใหเหล็กกินวันละ 3 เวลา B) Descending urethral transmission E) ฝากครรภตามปกติ C) homogenous spreading Exercise for National License Part II : March 2009 By NCTMS2009 หากมีขอเสนอแนะ ติชม เชิญแนะนําไดที่ www.nctms.in.th/webboard 17
  • 19. สหพันธนสิตนักศึกษาแพทยแหงประเทศไทย ิ 12. แมอายุ 48 ป พออายุ 50 ป มีลูกผิดปกติทาง 16. หญิงตั้งครรภมารพ.ดวยอาการปวดกลามเนื้อ มี โครโมโซม chromosome 44+XX, 21 trisomy พอ รอยฟกช้ําและจ้ําเขียวที่เบาตา หนาอก แขน ทอง แมอยากมีลูกคนที่ 2 ทองตอไปแนะนําอยางไร และตนขา ทั้งรอยเกาและใหม คิดวาเกิดจาก สาเหตุ A) ฝากครรภตามปกติ ใดมากที่สุด B) ตรวจโครโมโซมเด็กทารก A) ITP C) แนะนําผสมเทียมโดยใชอสุจิผูอื่น B) coagulopathy D) แนะนําทํา vitro fertilization โดยใชไข C) liver disease ของผูบริจาค D) domestic violence E) ไมตองตั้งครรภ 17. แมมีหมูเลือด Rh negative พอเปน Rh positive 13. 47,XX+21จะใหคําแนะนําอยางไรกับผูปกครอง หากตั้งครรภควรสงตรวจอะไรเพื่อเปนการลด โอกาส A) หามตั้งครรภ ในการเกิด hydrops fetalis B) ตั้งครรภืไดแตตองทํา amniotomy A) indirect Coombs’ test B) direct Coombs’ test 14. หญิง 50 ป postmenopausal bleeding, C) anti-D titer TVS: endometrial thickness 2 mm จะทําอะไรตอ A) endometrial biopsy 18. หญิง 27 ปตั้งครรภครบกําหนด EFW 3200 g B) F/C เบงคลอดมา 1 ชั่วโมง PV พบวา fully dilated, station+1, mento-posterior position ตองทํา 15. หญิงตั้งครรภ 20 สัปดาห มี greenish yellowish อยางไรตอ vaginal discharge, vaginal แดง cervix นุมสีมวง A) manual rotation จงใหการรักษา B) F/E A) oral doxycycline C) V/E B) oral tinidazole D) normal Vg delivery C) oral clotrimazole D) oral fluconazole 19. ผูปวยหญิง S/P C/S post op day 4 มีไข ตรวจ E) vaginal clotrimazole รางกายพบรอยแตกที่รูไหม มี fluctuation จะทําอะไร A) ตัดไหมออก B) ตัดไหมออกและให ATB C) ตัดไหมออกและแหวกแผลลึก D) ATB alone Exercise for National License Part II : March 2009 By NCTMS2009 หากมีขอเสนอแนะ ติชม เชิญแนะนําไดที่ www.nctms.in.th/webboard 18
  • 20. สหพันธนสิตนักศึกษาแพทยแหงประเทศไทย ิ 20. หญิงวัยรุนมาตรวจเรื่องโดนขมขืน ตรวจรางกาย 23. คนทอง term ถูก augmentation ดวย พบ old tear ที่ 6 นาฬิกา มี sperm ไมมีหาง ปริมาณ syntocinon 20d/min ตอมาพบวามีอาการเจ็บครรภ เล็กนอยที่ posterior fornix มีรอย kiss bite ที่ลําคอ มาก ตรวจร า งกายพบรอยคอดบริ เ วณ มดลู ก จงใหความเห็น เ ห ตุ ก า ร ณ ใ ด จ ะ เ กิ ด เ ป น ลํ า ดั บ ต อ ไ ป A) ตรวจพบหลักฐานวาโดนขมขืน A) uterine rupture B) ตรวจพบหลักฐานวาผานการมีเพศ B) abruptio placenta สัมพันธ C) retained placenta C) ตรวจพบหลักฐานวาผูปวยถูกกระทํา  D) uterine inertia ชําเรา E) uterine eversion 21. หญิงวัยรุนทองแลวมีเลือดออก มาพบแพทย ตองการใหทําแทง เนื่องจากเคยตรวจกับแพทยทา 24. จากขอที่แลว จงใหการรักษาเบื้องตน นอนแลวแพทยบอกวาทารกสุขภาพไมแข็งแรง ทาน A) off syntocinon จะทําอยางไร B) set C/S emergency A) ทําแทงให เนื่องจากทารกในครรภไม C) manual removal of the placenta แข็งแรง D) increase syncinon level B) ทําแทงให เนื่องจากถาไมทําก็คงมีคน E) ARM with C/S emergency ทําใหอยูดี C) ไมทําให เนื่องจากไมมีขอบงชี้ในการยุติ  การตั้งครรภทางการแพทย 22. จากการตรวจภายในเพื่อประเมินชองทางการ คลอด ได diagonal conjugate 11 cm, Interspinous 9cm, intertuberous 8cm, subpubic angle 105 มีความผิดปกติที่ใด A) pelvic inlet B) midplane C) pelvic outlet D) pelvic inlet + midplane E) midplane + pelvic outlet Exercise for National License Part II : March 2009 By NCTMS2009 หากมีขอเสนอแนะ ติชม เชิญแนะนําไดที่ www.nctms.in.th/webboard 19
  • 21. สหพันธนสิตนักศึกษาแพทยแหงประเทศไทย ิ Pediatric 4. เด็กหอบเหนื่อยตรวจรางกายพบ neck vein 1. เด็ก 10 ป เปนโรคหืด เปนๆ หายๆ รักษาไม engorge, lung = fine crepitation at base both สม่ําเสมอ วันนี้หอบเหนื่อยมากขึ้น PE : good lung มีขาบวม จําไมไดแลว consciousness,no cyanosis, prolong expiration A)heart failure time with wheezing both lungs ไดรับการรักษา เปน nebulized salbutamol ไป 2 dose แลวอาการ 5. เด็ก 3 ป ปวดทอง PE:Confused , diaphoretic ยังไมดีขึ้น ควรทําอะไรตอ profound sweating , drooling , meiosis ถามวาให A) nebulized adrenaline การรักษาอยางไร B) aminophylline infusion A) Naloxone C) IV corticosteroid B) Atropine D) nebulized corticosteroid C) Ethanol E) nebulized ipratropium bromide D) Flumazil E) Diphenyramide 2. เด็กทารก อายุ 2 วัน คลอดที่บาน มารพ.ดวย อาการตัวออน ไมรอง  6. เด็ก 8 ป มีไข คลื่นไส อาเจียน ปวดทองมา 3วัน PE : flaccid, flobby, RR 10/min, HR 50/min การ PE: epigastrium pain รักษาใดควรทําเปนอันดับแรก Lab: serum : Na 145 K 3.0... A) Glucose UA: ketone 4+ B) Arenaline ถามวาควรทําอยางไรในชั่วโมงแรก C) NSS A) 0.45 % NaCl 20 ml/kg D) Chest compression B) 0.9% NaCl 20 ml/kg E) PPV C) 0.45% NaCl 10 ml/kg + KCl 20 mEq D) 0.9% NaCl 10 ml/kg + KCl 20 mEq 3.เด็กอายุ 6y โดนสุนัขไมมีเจาของกัด ปกติไดรับ E) RLS vaccine ครบมาตลอดครั้งสุดทายตอน 5y จะให vaccine อะไร 7. เด็กชายอายุ 5 ป มาดวยกลามเนื้อออนแรง A) ไมให เปนมากบริเวณ deltoid and hip flexor เกิดจาก B) rabies day 0,3 สาเหตุใด C) rabies day 0,3,7 A) beri beri D) rabies day 0,3,7,30 B) muscular dystrophy E) rabies vaccine + TT Exercise for National License Part II : March 2009 By NCTMS2009 หากมีขอเสนอแนะ ติชม เชิญแนะนําไดที่ www.nctms.in.th/webboard 20
  • 22. สหพันธนสิตนักศึกษาแพทยแหงประเทศไทย ิ 8. เด็ก 2 ป ซีด มีจุดแดงที่ขา มา 2 wks ตรวจ A) สงตรวจการไดยิน รางกายพบ moderate pale , petichiae at leg and B) สง Consult หมอตา RUQ mass about 3x5 cm. firm, hard C) สงแพทยเฉพาะทางทันที consistency CBC: Hct 21%, plt 15,000, WBC D) แกไขภาวะโภชนาการกอนแลวจึงRefer 3,600 มี N 30%, L 50% ,M 6% จง diagnosis E) คอยมาผาตัดตอนอายุครบ 1 ป A) wilm tumor B) neuroblastoma 11. เด็ก 5 ป มีอาการเหมอลอย เปนพักๆ ตรวจพบ C) hepatoblastoma 3 Hz. spike wave เมื่อทํา hyperventilation จะให D) non-Hodkin lymphoma ยาใด E) acute lymphoblastic leukemia 12. เด็กหายใจเสียงดัง ดีขึ้นเวลานอนคว่ํา 9.เด็กหญิง 2ขวบ Wt แรกคลอด 2800g. long จง diagnosis 47cm. มาที่ Well baby clinic ผูปวยแข็งแรงดี ตรวจ A) larlyngomalacia การเจริญเติบโตตาม Growth curve อยูที่ P3 ทั้ง B) epiglottic cyst สว นสู ง และน้ํ า หนั ก พ อสู ง 156cm. แม สูง 149cm. แมมีประจําเดือนครั้งแรกตอน 13y ทานจะทําอยางไร 13. เด็กชายไทยอายุ 7 ป มีอาการไข น้ํามูกสีเขียวขน A) เปนภาวะปกติเพราะเปนผลจาก ไหล มีอาการเสมหะเขียวเปนๆหายๆ พันธุกรรม PE : erythematous swelling turbinate, B) ใหกินอาหารที่มีโปรตีนและพลังงานสูง postnasal drip, mucopurulent discharge C) สงปรึกษากุมารแพทย ผูปวยรายนี้นาจะเปนอะไรมากที่สุด D) ตรวจระดับ hormone A) Sinusitis E) ตรวจ bone mass B) Bronchiolitis C) Allergic rhinitis 10. ผูปวยเด็ก 5 เดือน มาดวยมีกอนที่คิ้วขวา ใหญ มาบดบังตาขวา เด็กยังชันคอไมได เรียกแลวไมหัน 14. เด็กแรกเกิดดูดนมแลวเขียว หายเขียวเวลารอง X-ray พบวาไมมี Frontal sinus และกระดูก Frontal จงใหการวินิจฉัย เชื่อมกับ Ethmoid , Body weight – P25 ควรทํา A) Coanal atresia อยางไรตอ B) TE fistular C) PDA Exercise for National License Part II : March 2009 By NCTMS2009 หากมีขอเสนอแนะ ติชม เชิญแนะนําไดที่ www.nctms.in.th/webboard 21
  • 23. สหพันธนสิตนักศึกษาแพทยแหงประเทศไทย ิ 15. เด็ก 4ป ไข ไอ หอบ กอนหนานี้มีน้ํามูกใสๆไหล A) Arthrocentesis PE:Trachea shift to Left , decrease Rt.breath B) Rt. Knee x-ray sound CXR:hazzeness Rt.lung C) Bone aspiration ควรสงตรวจอะไรเพื่อชวย Dx. D) ESR A) Tuberculin test E) Bone scan B) … C) Thoracocentesis 20. เด็ก 17 ป ประสบอุบัติเหตุกระแทกขณะเลน D) … ฟุตบอล 1 สัปดาหกอนมารพ. E) Nasopharyngeal culture 4 วันกอนมารพ. ปวดเขา มีไข 1 วันกอนมารพ. ปวดจนเดินลงน้ําหนักไมได 16. เด็กแรกคลอดอาเจียนเปนเลือด หลังคลอด 6 hr จะสงตรวจอะไรเพิ่มเติม A) Apt’s test A) Film knee B) striae tear B) Hemoculture C) plt C) Arthrocentesis D) Serum uric acid 17.เด็กชายมาดวย multiple purpura ที่ลําตัว แขน ขา และรอยเตารีด พอบอกวาลมลงโดนเตารีด ควร 21. เด็ก 12 ป นน.ขึ้นผิดปกติตั้งแตอายุ 2 ป ทําอยางไร PE : obesity, acanthosis nigricans A) ซักจิตเวชในครอบครัว นาจะตรวจพบสิ่งใด B) สง coagulogram A) Cortisol สูง C) แยกจากพอแม แลวคอยซักประวัติ B) Hyperinsulinemia C) Growth hormone สูง 18. ผูปวยเด็กมี acanthrosis nigrican ขอใดถูก D) Hypothyroidism A) insulin เพิม ่ E) Hyperuricemia B) cortisol เพิม่ C) thyroid เพิ่ม 22. เด็กชายมาดวยอาการเจ็บหู ไดรับการวินิจฉัย เปน Otitis media ไดยา Amoxycillin มาทาน 2 วัน 19. เด็กชาย 10 ป ปวดเขาขวา มีไข มา…วัน อาการไมดีขึ้น ยังเจ็บหู จะ management อยางไร PE : BT 39 c, swelling at Rt. Knee and proximal A) oral amoxicillin tibia, Rt. Inguinal LN enlarged and tendered B) oral amoxy-clavurunic acid จะสงตรวจอะไรเพื่อวินิจฉัย C) tympanoplasty Exercise for National License Part II : March 2009 By NCTMS2009 หากมีขอเสนอแนะ ติชม เชิญแนะนําไดที่ www.nctms.in.th/webboard 22
  • 24. สหพันธนสิตนักศึกษาแพทยแหงประเทศไทย ิ 23. เด็ก 9 เดือน พูดเปนคําๆ ไมมีความหมาย เกาะ A) meningitis ยื น แต เ ดิ น ไม ไ ด ยอมให ทุ ก คนอุ ม เวลาขอของไม B) hypoglycemia ยอมให เด็กมีความผิดปกติดานใด C) polycythemia A) speech D) perinatal asphyxia B) walk E) intraventricular hemorrhage C) separation anxiety D) to give toy 27. เด็กอายุ 1 ป มีไข 1 วัน ซึมลง PE: BT 39.5°C, BP 80/60 PR 140 RR 42, petechiae with 24. เด็กมีอาการปวดหัวมาก n/v ไข มีประวัติวาไป purpura at lower extremities จงใหการรักษา เลนน้ําในลําคลองมาดวย ตรวจรางกาย stiff neck เบื้องตน positive นาจะเกิดจากเชื้ออะไร A) dextran A) Nae B) dopamine B) H. influ C) NSS C) Strep pneumo D) platelet E) FFP 25. เด็ก 2 ป มาwell baby clinic เคยได BCG 1 เข็ม HBV 3 เข็ม ( DDT,OPV 3 เข็ม ) JE 2 เข็ม MMR 1 28. เด็กชาย 3 ป มารพ.ดวยอาการปวดทอง เข็ม ถามวาครั้งนี้ใหอะไร PE : confuse, diaphoretic, profuse sweating, A) JE drooling and miosis. B) JE, Hib การรักษาที่ เหมาะสมที่สุดไดแก C) OPV, DPT,JE A) ethanol D) OPV, DPT, Hib B) atropine C) naloxone 26. ทารกแรกคลอด GA 41 wk BW 3400 g มี D) haloperidol meconium stained AF APGAR= 2,5 ที่ 1 และ 5 E) dimenhydramine นาที 10 ชั่วโมงตอมามีอาการชักเกร็งทั้งตัว ตรวจ เลือด glucose 50 Hct 58% คิดวาอาการชัก เกิด จากอะไร Exercise for National License Part II : March 2009 By NCTMS2009 หากมีขอเสนอแนะ ติชม เชิญแนะนําไดที่ www.nctms.in.th/webboard 23
  • 25. สหพันธนสิตนักศึกษาแพทยแหงประเทศไทย ิ 29. เด็กอายุ 3 วัน อาเจียนน้ําสีเขียว กินนมแมตั้งแต 32. เด็กชายอายุ 1 ป มีไขมา 2 วัน ซีด ปสสาวะสีเขม วันแรก มารดามี PROM 10 ชั่วโมงกอนคลอด ตรวจ กินยาลดไขกอนมารพ. รางกายทองอืดเล็กนอย ไมซึม จะinvestigate อะไร PE: moderate pale, mild icteric sclera, A) stool exam tachycardia, no hepatosplenomegaly. B) CBC CBC: Hct 24% WBC 11500 (N 76% L 21% C) H/C M3%) plt. 470000 D) plain film abdomen PBS: bite cell, basket cell, anemia จงใหการวินิจฉัย 30. ในรพ.ชุมชน เด็ก 8 เดือนมาดวย clinical A) Hb H disease meningitis ชัก ทําอยางไร B) G 6 PD deficiency A) แจงพอแมกอนเพื่อ LP C) PNH B) LP ไดเลยเพราะวาเปนเรืองเรงดวน ่ D) AIHA C) สงรพ.จงหวัดใหเร็วที่สุด E) hereditary spherocytosis D) CT brain E) เจาะเลือด H/C แลวให ATB 33. เด็กหญิง 6 ป หกลมมีแผลที่หนาแขงขวา 1 สัปดาหกอน ทําแผลเองที่บาน 3 วันกอนเริ่มมีไข สูง 31. เด็ก 9 เดือน ยังไมเดน เกาะยืนได พูดเปนคําไมมี แผลมี ห นองไหล 6 ชม.ก อ นมีเ จ็ บหน า อก เหนื่ อ ย ความหมาย ยอมให ทุ ก คนอุ ม ฟ น ยั ง ไม ขึ้ น ไม ใ ห หอบ บวม นอนราบไมได ของเล น เวลาขอ เด็ ก คนนี้ มี พั ฒ นาการด า นใดผิ ด PE : BT 39.8 BP 70/45 PR 140 RR 40 ปกติ no cyanosis, engorged neck vein, distant heart A) language sound, crepitation both lungs, liver 3 cm BRCM B) walking จงใหการรักษาเบื้องตน C) stranger anxiety A) pericardiocentesis D) teeth impaction B) dopamine E) don’t give toy when asked C) ET intubation D) CPR E) NSS Exercise for National License Part II : March 2009 By NCTMS2009 หากมีขอเสนอแนะ ติชม เชิญแนะนําไดที่ www.nctms.in.th/webboard 24